Vous êtes sur la page 1sur 91

Briefly enumerate the important sources of recruitment.

Answer :

Recruitment refers to the process of searching and attracting the required personnel for a job.
In other words, it is the process of finding the potential candidates and instigating them to
apply for the job. The following are the two important sources of recruitment.
i. Internal sources: Internal sources of recruitment refer to the sources that are within the
organisation. That is, through internal sources the jobs are filled up from inside the
organisation. It can take the form of transfers and promotions. Through transfers, the job of a
specific profile is filled by shifting a suitable person working in another department of the
organisation to the concerned department. Similarly, through promotions, higher position job
vacancies in the organisation are filled by promoting the lower level employees.
ii. External Sources: External sources of recruitment refer to the sources of recruitment that
are outside the organisation. Through external sources the jobs in an organisation are filled by
bringing in new people. For example, one of the external sources of recruitment is 'direct
recruitment' which involves putting up a notice board outside office and then following the
recruitment process on a specified date. Similarly, placement agencies work as an external
source as they act as a match maker for the job seekers and job providers.
Answer needs Correction? Click Here
Q2 :
Define the staffing process and the various steps involved in it?

Answer :

Staffing process refers to the procedure of filling the vacancies and keeping them filled. It
focuses on timely fulfillment of the human resource required within the organisation.
According to 'Theo Haimann', 'staffing pertains to recruitment, selection, development,
training and compensation of subordinate managers. This definition includes the steps
involved in the staffing process. Following is a brief description of various steps involved in
staffing.
(i) Estimation of the Required Manpower: The process of estimating manpower requirement
is the basic step in the process of staffing. It refers to knowing the number and the kind of
persons that are required in the organisation. Estimation process involves two steps-workload
analysis and workforce analysis. Work load analysis implies an estimation of the number and

the kind of persons required for various jobs. On the other hand, workforce analysis implies
an estimation of the existing persons. The two-step analysis reveals whether there is any
overstaffing or understaffing in the organisation and thereby, forms the basis of the staffing
process. For example, a situation of understaffing would imply that more personnel are
required to be appointed and a situation of overstaffing would imply that some of the existing
personnel need to be removed.
(ii) Recruitment/Searching: It refers to enlisting and searching the suitable candidates for the
job. It is the process of searching the appropriate people for the job and influencing them to
apply for the job. Recruitment creates a pool of prospective candidates for a job. It involves
searching through various sources that includes internal sources (transfers and promotions)
and external sources (advertising and placement agencies). It forms the basis for the selection
process by attracting the aspirants towards the organisation.
(iii) Selection: Selection involves choosing the right candidate out of the gathered pool of
aspirants which is created after recruitment process. It is a rigorous procedure and comprises
of series of tests, interviews, etc. The candidates who are not able to negotiate the selection
process are rejected. It ensures that only the competent and the best ones are selected for the
job. It is a highly important process as it forms the basis for working efficiency of the
organisation.
(iv) Induction and Placement: Once the selection is done it becomes important to make the
selected employees familiar with the working environment of the organisation. Induction
involves giving a brief overview about the workplace, introducing them to other employees
and the managers and making them comfortable with the work environment. Placement refers
to occupying the position by the employee for which he has been selected.
(v) Training and Development: Next step in the staffing process is training and development
of the employee. Both the process emphasise on improving the employee competence.
Training is a process of increasing the employee's capabilities and skills required for
performing the job. Development, on the other hand, focuses on the overall growth of the
employee by enhancing his thinking and understanding capabilities. They help the workers to
upgrade their knowledge and increase their efficiency. Moreover, training and development
motivates the workers and provides them opportunities for growth and career development.
(vi) Appraisal: Performance appraisal is an important aspect of any organisation as it helps in
evaluating the work of the individuals. Appraisal implies assessing the performance of the
employee against certain predetermined standards. In addition, under performance appraisal
the superior provides proper feedback to the employee so that right measures can be taken for
increasing the working efficiency.
(vii) Promotions: Every employee needs encouragement and motivation for continuing the
work with right efficiency. Promotion of the employee in terms of position, pay, etc. helps in
providing job satisfaction to the employee and encourages them to realise their potential.
Promotions serve the long term interests of the employees.

(viii) Compensation: Worth of the job is an important aspect to determine. All organisations
need to establish the right pay or salary for each job. Compensation entails the price of a job
along with the rewards that the employee deserves. Compensation provided to the employee
can be in direct terms (that is, wages and salaries) as well as in indirect terms (such as
insurance, bonus, etc.
What is meant by recruitment? How is it different from selection?

Answer :

Recruitment refers to the procedure of finding and stimulating the required candidates to
apply for a particular job. As against this, selection is the procedure of screening and
choosing the required candidates out of the gathered pool.
The following points highlight the difference between recruitment and selection.
Basis of Difference
Meaning

Sequence
Employment Contract

Characteristic

Recruitment

Selection

Recruitment refers to the process Selection refers to the process of


of finding and instigating the
choosing the right candidate out of the
required personnel for a job.
gathered pool developed at the time of
recruitment.
In the staffing process,
In the staffing process, selection is at the
recruitment is at the second
third stage and succeeds recruitment.
stage.
The candidates gathered under
The candidates who successfully
recruitment are not offered any
complete the selection process are
employment contract from the
offered an employment contract by the
organisation.
organisation containing such information
as date of joining, terms and conditions,
etc.
Recruitment process involves
Selection process involves choosing only
attracting as many persons as
the appropriate candidate and rejecting
possible for the job.
the non-suitable ones.

Answer needs Correction? Click Here


Q4 :
Explain the procedure for selection of employees.

Answer :

Selection is a procedure to choose the appropriate candidates out of the numerous aspirants. It
is a rigorous process as it involves various stages of tests and interviews. Selection process
aims at getting the best out of the recruited pool so that work efficiency can be maximised.
Following are the steps involved in the selection process.
(i) Screening: To begin with, it is necessary to eliminate those candidates who do not fulfill
the basic criteria and qualification required for the job. This is done through preliminary
screening by examining the information provided in the application forms. These forms may
be in the form of online application or paper applications. Along with this, preliminary
interviews can also be conducted to bring out the facts which are not written in the
application forms and thereby, help in rejecting the misfits.
(ii) Test: Test is a mechanism that helps in judging certain characteristics of the aspirants.
These characteristics can be regarding skills, knowledge, intelligence, etc. Various forms of
testing are Intelligence test, Aptitude test, Personality test, Trade test and Interest test. They
assess the individuals on different platforms and find out the suitable candidates for further
selection process. For example, personality tests judge the candidate for characteristics such
as maturity level, emotional control, etc. Similarly, trade test examine the skill and
knowledge of the individual.
(iii) Personal Interview: Personal interviews are an integral part of selection process that
includes conversation of the candidates with the managers. Herein, in-depth conversations are
conducted to judge the suitability of the individual for the job. Sometimes, the individual may
also seek information regarding the organisation in such interviews.
(iv) Background Checks: Once the individuals pass through the interviews, some
organisations ask for references of other persons such as previous employers, other known
persons. From these references the organisation aims at verifying the information provided by
the candidate. Moreover, these references also act as a source of additional information of the
candidate.
(v) Selection Decision: After the tests and interviews are completed, the final selection is
done. The appropriate candidates are chosen by the concerned managers.
(vi) Medical Examination: Few organisations emphasise on conducting a medical
examination to check the health of the candidates. The job offer is given after they are
declared fit by a medical practitioner.
(vii) Job Offer: Next step is offering the job letter to the selected applicants. It refers to an
appointment letter given by the organisation to confirm that the individual has been selected.
Such letter generally comprises of such information as date of joining and the time of
reporting.

(viii) Employment Contract: Once the job letter is given, the candidates have to fill-up some
forms that are necessary for the future references. One such form is attestation form that
holds information about the candidates which are attested by him/her. Another document is
the contract of employment. It comprises of details such as salary, date and terms of joining,
leave rules, working hours, allowances, etc.
Answer needs Correction? Click Here
Q5 :
Define training. How is it different from education?

Answer :

Training refers to the enhancement of skills, abilities and competence required for a specific
job profile. On the other hand, education refers to the process of improving the overall
knowledge and understanding of the employees. The following points highlight the difference
between training and education.
Basis of
Difference
Meaning
Scope

Focus

Training
Training implies increasing the
skills and abilities required to
perform a specific job.
Training is a narrow concept and
relates to development of
understanding relating to only a
specific job.
It focuses on the goals of the
organisation.

Education
Education implies increasing the
overall knowledge and understanding
of the employees.
Education is a wider concept and
relates to the overall development of
understanding.
It focuses on the individual goals.

Answer needs Correction? Click Here


Q6 :
What are the advantages of training to the individual and to the organisation?

Answer :

Training is an important aspect of any organisation. It aims at building the skills and abilities
of the individuals to perform a job. Training is an essential part of the job which attempts at

improving the aptitude and knowledge of a person as per the job requirement. With the ever
changing business environment, the complexity of the jobs has increased. Thereby, training
has become all the more essential. The following are the highlighted benefits of training to
the employee and the organisation.
Benefits to the Employee
(i) Better Career Opportunities: Training enhances the skill and knowledge of the employees
and thereby, helps in improving their career prospects.
(ii) Earn More: By helping the individual to learn more it assist them to earn more. Training
enhances the individuals understanding and knowledge. Thereby, it improves their efficiency
and performance. As a result, it gives them a chance to earn more.
(iii) Less Accident Prone: Training attempts to make the employees more adept and efficient
in handling the machines. Jobs that require the employees to work with complex machines
and in more accident-prone areas need to emphasise strongly on training and development, as
it helps them in taking precautionary actions. Thus, it makes the employee less prone to
accidents.
(iv) Self-Confidence: Development and training direct the individuals to work more
efficiently that in turn builds up required morale within the individuals. It increases their selfconfidence and self-reliance thereby, contributing to higher job-satisfaction.

Benefits to the Organisation


(i) Less Wastage: Training is a concept that involves systematic learning of work. It is a more
efficient way of learning than any hit and trial method. This leads to less wastage of time and
money. Thereby, it helps in optimum utilisation of resources.
(ii) Higher Profits: By developing the individuals and making them more efficient, training
enhances their productivity in work. Due to increase in both qualitative and quantitative
productivity, the profits of the company rise.
(iii) Managerial Efficiency: Training prepares the employees to face new challenges in
different situations. This helps them in building effective responses to various situations.
Training also equips the future managers to take precautionary step in case of emergency.
(iv) Reduced Absenteeism: Effective training helps in boosting the morale of the employees.
Thereby, it helps in reducing the rate of absenteeism and employee turnover.
Answer needs Correction? Click Here
Q7 :
Distinguish between training and development.

Answer :

Training and development are related yet distinct concepts. Though both the concepts focus
on improvement of an individual but the perspectives are different. Training on one hand
refers to providing the skills and abilities for a particular job. Development on the other hand,
refers to the concept of growth of an individual as a whole.
The following points highlight the difference between training and development.
Basis of Difference
Meaning

Focus
Scope

Training
Training refers to the process of
enhancing the skills and competence
of an employee that are required to
perform a specific job.
The focus of training is the specific
job requirement and is thereby, joboriented.
Training is narrow in scope and
focuses on how one can become more
efficient in the intended job.

Development
Development refers to the process
of overall growth of an employee.
The focus of development is overall
growth and is thereby, careeroriented.
Development is wider in scope and
focuses on the overall personality
development of the employee.
Training is a part of development.

Answer needs Correction? Click Here


Q8 :
The staffing function is performed by every manager and not necessarily by a separate
department. Explain.

Answer :

Staffing refers to the function of finding the right people for the right job. It is an important
aspect of management as it ensures that the right people with the needed qualifications are
chosen for the work.
It ensures that the vacancies are filled and always remain occupied. Staffing involves various
stages including recruitment through various sources, right selection out of the applicants,
placement and orientation, training of the employees, etc. Managing the human element is a
requisite function of any organisation as it serves the basis of the level of efficiency of the
organisation. Thus, success of any organisation depends on how effectively this function can
be performed and to what extent the productivity can be increased with efficient staffing.

Nowadays, it is seen that organisations that are large in size, maintain a separate department
for dealing with this function. This department is generally known as 'Human Resource
Department'. Under this department, the various tasks of staffing are performed by
specialised managers. Along with staffing they also take care of the labour grievances.
Human resource management thus act as a link between the workers and the managers.
However, staffing does not necessarily require a separate department and rather, is a function
to be performed by every manager himself. Staffing forms an essential part of management.
Managers here play an important role as they make sure about the optimum utilisation of the
manpower. It is their task to watch if there is any vacancy and also, if any extra manpower is
needed. Subsequently, they should work on how the recruitment has to be done, what
qualifications are necessary for the applicants to apply for the job, and to what extent,
training is necessary for that particular job. As managers are more closely related to the
workers, they can work on the staffing process more efficiently. They can set the required
qualifications for a particular job according to their need and work. For example, if a
manager needs two subordinates to work under him then he'll be able to mention the
requirements more clearly. Similarly, they can perform the task of recruiting the personnel
more smoothly as they would know what kind of personnel he would require and through
what source he can gather an efficient pool of applicants, may it be internal or external
sources, depending on his needs. As the main seekers of the employees, they can carry out the
selection process of the right candidate in the best possible manner. Along with this,
managers can also perform other important aspects of staffing namely, appraisal, promotions
and career planning.
Thus, it can be said that staffing function is performed by every manager and not necessarily
by a separate department.
Answer needs Correction? Click Here
Q9 :
Why are internal sources of recruitment considered to be more economical?

Answer :

Internal sources of recruitment refer to the sources that are endogenous to the organisation,
that is, within the organisation. There are two ways of filling the jobs internally, namely
transfers and promotions. It has an advantage of being more economical than the other
sources of recruitment. Filling the jobs through internal sources is cheaper in terms of time as
well as money. On one hand, the time spent in the whole recruitment and selection process is
diminished to a large extent through internal recruitment. On the other hand, it lowers the
cost by saving the expenditure on advertisement and other related processes. Along with this,

the money spent on training of the new employees is also curtailed. Thus, in contrast to other
sources, the internal sources of recruitment are more economical in nature.

Answer needs Correction? Click Here


Q10 :
What is the importance of staffing function in today's environment?

Answer :

Staffing is an important function of management as it takes care of the manpower


requirement of any organisation. In today's environment with rapid changes taking place in
technology, size of the organisations, etc. finding the right people for the job becomes critical.
In such a scenario, proper staffing process plays an important role in the organisations.
Following are the highlighted benefits of staffing in the current world scenario.
(i) Finding Competent Personnel: Staffing helps in finding and choosing the right personnel
required for a job.
(ii) Improves Efficiency: By ensuring that right people are placed for right jobs, the overall
efficiency and performance increases.
(iii) Growth of the Organisation: It ensures survival and growth of the organisation by
appointing efficient and competent employees for various jobs.
(iv) Optimum Utilisation of Human Resources: Through proper manpower planning,
staffing prevents over-utilisation or under-utilisation of manpower. In addition, it avoids
interruption in working efficiency by suggesting, in advance if there is any unfilled job.
(v) Job Satisfaction: Compensation and fair rewards given to the employees provide them
self-confidence and job-satisfaction. It encourages them to work diligently and give their best
to the organisation.

Chapter 7

Q1 :
Distinguish between leaders and managers.
Answer :

The following points highlight the difference between a manager and a leader.

Manager

Basis of Difference
Existence
Objective

Power and Authority

Interrelationship
Functional Spread

A manager can exists only in a formal


organisation.
A manger focuses towards influencing
the behaviour of employees towards
achievement of the goals of the
organisation.
A manager holds formal authority and
can exercise power and control to
influence the behaviour of the
employees.
A manager can be a leader as well.
The functions of the manager are
widespread and include planning,
organising, staffing and directing.

Leader
A leader can exists in formal as
well as informal organisation.
A leader focuses towards the
satisfaction of the individual and
group goals of his followers.
A leader does not command
formal authority and can only
influence the behaviour of the
followers through such means as
trust and faith.
A leader may not be a manager.
Functions of a leader are limited
to directing.

Answer needs Correction? Click Here


Q2 :
Explain the principles of directing?
Answer :

Directing as a function of management is quite complex. To help in the directing process,


certain principles have been developed. The following are the principles of directing.
1. Maximum Individual Contribution: According to this principle, a manager should use
such directing techniques that induce the workers to perform to the best of their capabilities.
It should encourage the workers to work towards the goals of the organisation. That is, each
individual should contribute the maximum towards the organisational goals. For example,
suitable incentive and motivation techniques can be used by the managers to urge the
employees to perform better.
2. Harmony of Objectives: Often the objectives of an individual diverge from the overall
organisational objectives. For example, an individual's focus may be on earning greater
income while the organisation aims at increasing the production. In such a case, directing
should work towards converging the individual goals with the goals of the organisation.
3. Unity of Command: As per this principle, an individual should receive commands and
instructions from only one superior. If a worker receives orders from more than one superior,
it creates confusion and clashes that leads to delay in the work. For example, the worker

might get confused whose order to follow. Moreover, it might also happen that a conflict is
created between the two superiors.
4. Appropriateness of Direction Technique: According to this principle, the direction
technique to be used should be appropriately selected. It should suit the need and attitude of
the employees. For example, one employee might get motivated by a praise while other
requires monetary incentive. Thus, the manager should use suitable directing techniques on
requirement basis.
5. Managerial Communication: Effective communication plays a key role in directing. The
instructions and commands given by the superior must be clear and easily understandable by
the subordinates. In addition, the subordinates must also be able to communicate with the
superiors in an unhesitant manner. They must be able to express freely their feedbacks and
suggestions. Thus, an effective two way communication must take place between the superior
and the subordinates.
6. Use of Informal Organisation: The manger should realise the existence and importance of
informal organisation. He must strategically use them. For example, informal communication
can be used to divulge true and real feedback on policy matters.
7. Leadership: Leadership is an important element of directing. A manager must be able to
bring out the capabilities of the employees. He must be able to influence the behaviour of the
employees such as to boost the willingness of the employees to work towards the
organisational goals. In addition, he must also be able to work effectively towards the
satisfaction of the individual goals of the employees.
8. Follow Through: According to this principle, a manager's role is not just limited to giving
instructions to the subordinates rather, he should continuously review the implementation of
the instructions. Through a proper follow up he must ensure that the commands are properly
followed and implemented. If required suitable corrective actions must also be taken.
Answer needs Correction? Click Here
Q3 :
Define Motivation.
Answer :

Motivation refers to a process of inducing and stimulating an individual to act in certain


manner. In the context of an organisation, motivation implies encouraging and urging the
employees to perform to the best of their capabilities so as to achieve the desired goals of the
organisation. In other words, it refers to driving the individual psychologically so as to induce
his willingness to work and perform better. In an organisation motivation can take various
forms such as promotion, appraisal, recognition, etc. depending on the expectations and
desires of the employee. For example, an appraisal may act as a motivating factor for an

employee to improve performance. Similarly, for another employee praise from the senior
may motivate him to further improve the performance.
Answer needs Correction? Click Here
Q4 :
Explain the qualities of a good leader? Do the qualities alone ensure leadership success?
Answer :

It is said that to be a successful leader an individual must possess certain qualities. Some of
the qualities of a good leader are as follows.
1. Physical Attributes: People with good physical features such as height, appearance, health
etc. are attractive. A healthy and active person can himself work hard and efficiently and
thereby, has the capability of being looked up to. Thus, he can induce his subordinates as well
to work and perform better.
2. Honesty: A good leader should maintain high degree of honesty. He should be sincere and
should follow ethics and values. He should be an idol for others in terms of honesty, integrity
and values.
3. Intelligence: A leader must have a good presence of mind and knowledge. He should be
competent enough to effectively examine and solve the problems encountered in the course of
work. He must have the required intelligence to take proper decisions based on logic and
facts.
4. Inspiration: A leader should be a source of inspiration and motivation to others. That is, he
must be exemplary in terms of work, performance and values. He must be able to develop
willingness among the subordinates to work to the best of their capabilities.
5. Confidence: A leader should be high in confidence. He must also be able to maintain his
confidence in difficult situations as well. Only when a leader is confident himself, he can
boost the confidence of his subordinates.
6. Responsibility: A leader should command responsibility for the work and tasks of his
group. He should hold the responsibility of being answerable for the mistakes of his
subordinates. However, as a mark of encouragement he must share the credit of the success
with his subordinates.
7. Effective Communication Skill: A leader should be able to clearly express his ideas and
instructions clearly to the subordinates. On the other hand, a leader also forms the link
between the higher authorities and the subordinates. He should be able to effectively pass the
problems and suggestions of the subordinates to the seniors. Besides, he should also be a
patient listener and counsellor.

8. Ability to take Decisions: A leader should be able to take appropriate decisions based on
logic, facts and figures. Moreover, he should be confident enough to hold on to his decisions
and not get confused.
9. Social Behaviour: He should maintain a friendly and supportive behaviour with his
subordinates. He must be able to understand people and maintain good social relations with
them.
10. Dynamic: A leader must be dynamic and outgoing. He must be able to take up new
initiatives and break the old paradigms for the benefit of the organisation.
Though the above mentioned qualities are necessary for being a good leader, however, the
mere presence of these qualities does not ensure leadership success. In fact, no single
individual can possess all the qualities. However, a conscious effort must be made by the
managers to acquire them.
Answer needs Correction? Click Here
Q5 :
What is informal communication?
Answer :

Informal communication refers to the communication that flows without following the formal
defined path. An informal communication system is also known as grapevine. Under informal
communication, a piece of information flows in all directions without paying any heed to the
level or authority. It arises out of social interactions among the employees and then spreads
throughout the organisation. It can take the form of a rumour or a gossip. For example, a talk
over lunch regarding the attitude of a senior is an informal communication. It is difficult to
determine the source of such information. Moreover, the actual talk may even get distorted
and the final piece of information may come out to be much different from the original one.
Answer needs Correction? Click Here
Q6 :
Discuss Maslow's Need Hierarchy theory of motivation.
Answer :

Maslow's Need Hierarchy theory provides an understanding of the complex phenomenon of


motivation. According to Maslow, within each individual there exists a set of five needs that
can be arranged in a hierarchy. Knowledge of these needs helps the manager in understanding
the behaviour of employees. With identification of the needs of the employees, appropriate
motivation can be provided to them.

Assumptions
Maslow's theory is based on the following assumptions.
(i) People's need influences their behaviour.
(ii) Needs of the individuals can be arranged in a hierarchical order.
(iii) An individual can move to a higher level need only when the lower level need in the
hierarchy is satisfied.
(iv) Once a need is satisfied, an individual can be motivated only through the next higher
level need.
Theory
The following is the hierarchy of needs as given by Maslow.
1. Physiological Needs: Such needs comprise of essential requirements for sustenance of life.
They are at the top of the hierarchy. The fulfilment of these needs is necessary for survival.
For example, food, clothing, shelter are physiological needs. An employee requires a certain
basic salary for the satisfaction of these needs.
2. Security Needs: These needs relate to physical and economic security and well being. For
example, an employee desires job security, income stability, etc.
3. Belongingness Needs: Such needs comprise of the social needs of an individual such as
affection, acceptance, companionship, etc. as every individual yearns for social acceptance
and belongingness.
4. Esteem Needs: These needs include such elements as respect, dignity, recognition, etc. as
every individual wants to command respect and acknowledgement in the peer group.
5. Self-Actualisation Needs: This need refers to achieving what one aims or aspires. It is the
highest level need in hierarchy. For an employee such needs relate to growth, work
satisfaction, etc.
Maslow's theory is widely appreciated and is used as basis for motivation by the managers.
However, sometimes it can happen that needs of an individual do not follow the exact order
of hierarchy. Nevertheless an understanding of the needs as given by Maslow helps the
managers in practicing efficient motivation.
Answer needs Correction? Click Here
Q7 :
What are semantic barriers of communication?
Answer :

Semantic barriers of communication relate to the use or understanding of language.


Sometimes certain words, sentences or phrases are misinterpreted or misunderstood. In such
cases, effective communication is obstructed. Such barriers in communication that arise out
of ambiguity or difficulty in understanding of words and sentences are known as semantic
barriers. The following are some of the causes of semantic barriers.
i. Sometimes due to poor vocabulary or wrong use of words, the information may not be
clearly expressed.
ii. At times a word may have more than one meaning or two or more words may have same
pronunciation (such as idle and idol). In such cases, the correct interpretation of the word
remains ambiguous.
iii. In certain cases the proficiency of a language differs among the workers and the mangers.
In such cases, a translation of the information is required in the language which is
understandable to the workers. However, in the process of translation some of the words or
sentences may get misinterpreted. For example, in a translation of an instruction from English
to Hindi, the meaning of certain words might change.
iv. At times while giving out instructions the senior or specialist uses technical vocabulary
that might be difficult to understand for the subordinates.
Answer needs Correction? Click Here
Q8 :
What are the common barriers to effective communication? Suggest measures to
overcome them.
Answer :

Barriers in Communication
Sometimes the information that reaches the receiver is not in the manner that the sender had
intended. That is, at times there arises misunderstanding or misinterpretation of the
information as it is passed from the sender to the receiver. This creates barriers in the
effective flow of communication. Barriers to communication can be classified as follows.
1. Semantic Barriers: Semantic barriers of communication relate to the use or understanding
of language. Sometimes it happens that certain words, sentences or phrases remain
ambiguous or difficult to understand. Thereby, they are likely to get misinterpreted. Such
barriers in communication that arise out of ambiguity or difficulty in understanding of words
and sentences are known as semantic barriers. For example, sometimes while giving out
instructions the senior or specialist uses technical vocabulary that might be difficult to
understand for the subordinates. Similarly, at times two or more words have the same
pronunciation (such as access and excess), that results in confusion regarding the correct
interpretation of the word.

2. Psychological Barriers: Sometimes psychological factor such as frustration, anger, fright


may also obstruct effective communication. For example, out of frustration over a certain
matter, an individual's mind may be preoccupied and he may not be able to attentively grasp
the information given to him. Similarly, due to preconceived notions regarding a
conversation, an individual might derive conclusions even before the information is
completed.
3. Personal Barriers: Sometimes personal factors related to the sender or the receiver act as a
hurdle in communication. For example, often in formal organisations, superiors do not share
such information that they fear will harm their authority. Similarly, due to lack of trust on
their subordinates, they may not be willing to pay attention to the information provided by
them. In a similar manner, subordinates may lack the incentive to communicate freely with
the superiors. Thus, in such cases effective communication is hindered due to personal factors
pertaining to the sender and the receiver.
4. Organisational Barriers: In formal organisational structures, barriers to communication
arise due to such factors as authority, rules, regulations, relationships, etc. For example, if an
organisation follows long vertical chains of communication, it might result in delay in the
flow of information. Similarly, a highly centralised organisational structure obstructs free
communication.
Measures to overcome Barriers in Communication
The following are some of the measures that can be adopted to overcome various barriers of
communication.
(i) The communication should take place as per the understanding level and capabilities of
the receiver. That is, it must be ensured that the receiver is clearly able to understand the
information.
(ii) The language, tone and content of the information should be appropriately chosen. It
should be easily understandable and should not harm anybody's sentiments.
(iii) For the communication to be effective proper feedbacks must be taken from the receiver.
That is, he must be encouraged to respond during the conversation.
(iv) It must be ensured that the information is complete in all respect and nothing is left
ambiguous.
(v) The core idea of the communication must be clear between the sender and the receiver.
That is, it must be conveyed properly what the communication is about.
(vi) The sender of the information should also be a patient listener. He should be open to
communication from the other end as well.
Answer needs Correction? Click Here
Q9 :

Who is a supervisor?
Answer :

A supervisor refers to a person who directly oversees the activities of the workers. In the
organisational hierarchy he lies immediately above the worker. A supervisor performs the
following functions.
i. He is in direct contact with the workers and thereby guides and supports the workers. He
also helps in maintaining harmony and unity among the workers.
ii. He acts as a link between the manager and the workers. Through supervisor, on one hand,
the management communicates the ideas and policies to the workers and on the other hand,
the workers are able to communicate their problems and grievances to the management.
iii. Supervisor ensures that the workers work efficiently and as per the set targets.
iv. He provides the required knowledge and skill to the workers.
Answer needs Correction? Click Here
Q10 :
Explain different financial and non-financial incentives used to motivate employees of a
company?
Answer :

Financial Incentives
Financial incentives refer to direct monetary incentives offered to the employees to motivate
or reward people for better performance. The following are some of the financial incentives
used in the organisations.
i. Salary and Allowances: In every organisation salary and allowances given to the
employees forms the basic form of financial incentive. Regular raise in salaries and grant of
allowances acts as a motivation for the employees
ii. Performance Based Incentives: Often organisation offer monetary rewards for good
performance. This induces the workers to improve their efficiency and performance.
iii. Bonus: Bonus refers to the extra reward over and above the basic salary. It can take the
form as cash, gifts, paid vacations, etc. For example, some organisations grant bonus during
festival times such as Diwali bonus.
iv. Stock Option: Under this incentive scheme, the employee is offered the shares of the
company at a price lower than the market price. This instils a feeling of ownership and

belongingness in the employee and urges him to contribute towards the goals of the
organisation.
v. Sharing of Profit: Herein, the organisation shares a portion of the profit with its
employees. This encourages the workers to contribute actively towards the growth of the
organisation.
vi. Retirements Benefits: Many organisations offer certain retirement benefits to its
employees such as pensions, gratuity, provident fund, etc. This provides a sense of security
and stability to the employees.
vii. Fringe Benefits: Besides the basic salary an organisation may offer certain additional
advantages also to its employees such as housing allowance, medical allowance, etc.
Non Financial Incentives
Non-financial incentives refer to those incentives that focus on non-monetary needs of the
employees such as the social and psychological needs. The following are some of the nonfinancial incentives used in the organisations.
i. Position: Rise in status in terms of power, authority, responsibility provides a psychological
boost to the employees. For example, a promotion may satisfy the esteem and self
actualisation needs of an individual.
ii. Organisational Characteristics: Certain characteristics such as employee freedom,
recognition of performance, incentives and rewards play an important role in influencing the
behaviour of the employees. For example, if the employees get due recognition for their
performance, it encourages them to work more efficiently.
iii. Work Enrichment: Often, a challenging work endowed with greater responsibility and
requiring higher knowledge and skill enhances the interest of the employee. It provides the
employee prospects for personal growth. Thus, it proves to be a good source of motivation
for him.
iv. Career Opportunities: If the organisation is endowed with appropriate growth and career
opportunities for its employees, it strives then to perform better and thereby, climb the
professional ladder.
v. Job Security: An employee should have a certain extent of security regarding his
association with the organisation. Constant fear of losing the job hampers their efficiency.
However, a complete security can also result in loss of interest in work.
vi. Involvement: If an organisation allows the participation of the employees in the policy and
decision making matters, then it instils a feeling of belongingness in them and motivates them
to work towards the organisational goals.
Answer needs Correction? Click Here

Q11 :
What are elements of directing?
Answer :

Directing refers to the process of instructing, motivating, guiding and leading the people to
achieve certain goals and objectives. Directing involves the following four elements.
i.Supervision: Supervision implies the process of guiding and instructing the subordinates
towards achieving the desired goals. In other words, it implies overseeing the work of the
subordinates. Supervision ensures that work takes place as per the desired objectives. A good
supervision helps in improving the efficiency of the workers. Besides this, it also plays a key
role in maintaining harmony and unity among the workers.
ii.Motivation: Motivation implies encouraging and inducing the employees to perform to the
best of their capabilities so as to achieve the desired goals of the organisation. Motivation can
take various forms such as promotion, appraisal, recognition, etc. By satisfying the needs of
the employees, motivation helps in improving their performance. It provides a psychological
boost to the workers and drives their willingness to work. Moreover, it also helps in reducing
the turnover and absenteeism in the organisation.
iii.Leadership: Leadership implies influencing the behaviour of the employees in such a
manner that they willingly work towards achieving the objectives of the organisation.
Leadership plays a key role in the success of an organisation. Good leadership brings out the
capabilities and talents of the workers and thereby, boosts their confidence. They act as guide
to the workers and induces a feeling of initiative in them.
iv.Communication: Communication refers to the process of exchange of ideas, feelings,
facts, etc. among people. A smooth functioning of an organisation requires a good
communication. It fosters coordination among various departments and individuals in the
organisation. Communication forms the basis of management. Without good communication
network efficient management becomes difficult.
Answer needs Correction? Click Here
Q12 :
Explain the process of motivation?

Answer :

Motivation implies inducing and stimulating an individual to act in certain manner. The
following points explain the process of motivation.
i.Unsatisfied Want: The motivation process begins with an unsatisfied need of an individual.

ii.Frustration: As the want remains unsatisfied frustration builds up in the mind of the
individual.
iii.Drives: The frustration drives the individual to look out for alternatives to satisfy his need.
iv.Behaviour: Among the various alternatives he chooses one and starts behaving according
to it.
v.Satisfaction: After following a particular alternative for some time, he assesses if his need
is satisfied.
vi.Reduced Frustration: Once the need is satisfied, the frustration and tension of the
individual finally gets reduced.
For example, suppose an individual desires promotion. This makes him uneasy and he starts
looking out for alternatives through which he can earn a promotion. He may think of working
harder and improving his performance. After consistently working hard, he may get
recognition and the promotion that finally satisfies his want and reduces his frustration.
Answer needs Correction? Click Here
Q13 :
Explain the different networks of grapevine communication?
Answer :

Grapevine communication or informal communication refers to the communication that


arises out of social interaction among employees and spreads without following the formal
communication path. The following are the types of grapevine communication network.
i. Single Strand Network: In this network, the information spreads from one person to other
in a sequence. That is, one person communicates to another person who turn communicates to
some other person.
ii. Gossip Network: In gossip network, one person shares the information with many other
people.
iii. Probability Network: Under a probability network, an individual shares the information
randomly with other people. That is, the person is indifferent about who he shares the
information with.
iv. Cluster Network: In this network, information is first shared between two people who
trust each other. One of them then passes the information to some other person who in turn
shares it with another and so the information spreads.
Answer needs Correction? Click Here
Q14 :

The highest level need in the need Hierarchy of Abraham Maslow:


(a)Safety Need
(b)Belongingness Need
(c)Self Actualisation Need
(d)Prestige Need
Answer :

The highest level need in the need Hierarchy of Abraham Maslow is self actualisation need.
This need of an individual refers to achieving what one aims or aspires. For an employee in
an organisation, such needs relate to growth, work satisfaction, etc.
Answer needs Correction? Click Here
Q15 :
The process of converting the message into communication symbols is known as:

(a) Media
(b) Encoding
(c) Feedback
(d) Decoding
Answer :

The process of converting the message into communication symbols is known as encoding.
The communication symbols in the process of encoding can be in the form of pictures,
gestures, etc.
Answer needs Correction? Click Here
Q16 :
The communication network in which all subordinates under a supervisor communicate
through supervisor only is:

(a)Single chain
(b)Inverted V
(c)Wheel
(d)Free flow

Answer :

The communication network in which all subordinates under a supervisor communicate


through supervisor only is wheel communication network. In a wheel network, the supervisor
is at the centre of the communication network (wheel). The subordinates cannot communicate
among themselves, rather they need to communicate through the supervisor.

Chapter 8 : controlling

Short answerslong answers : Solutions of Questions on Page


Number : 234
Q1 :
Explain the meaning of controlling.
Answer :

Controlling refers to the function of evaluating and assessing the progress of the work done.
It involves setting a specific criteria or standards for the work and then comparing the actual
work with the set standards. It helps in finding the deviations from the set targets and thereby,
take the required corrective actions. It ensures that everything goes as per the plans adopted.
It also ensures full and efficient utilisation of resources. Controlling is an imperative
managerial function as it keeps a close check on the progress of work and thereby, forms the
basis for future actions and planning.
Answer needs Correction? Click Here
Q2 :
Explain the various steps involved in the process of control.
Answer :

Controlling is a systematic approach of managing and controlling the organisational actions.


The following are the steps involved in the controlling process.
(i) Setting Standards
Setting up of standards involves developing the benchmarks against which the actual
performance is to be measured. The standards can be set in qualitative as well as quantitative
terms. Qualitative benchmarks can be in the form of improving coordination in work, higher
goodwill or increased motivation level of employees, etc. For example, to improve the

motivation level among employees, standard can be set in terms of number of initiatives
taken. Quantitative benchmarks can be in the form of sales targets, units to be produced or
time to be spent on a particular action, etc. For example, in a shirt factory completing 10
pieces a day is a quantitative target. The standards that are set should be such that they
facilitate easy comparison.
(ii) Measuring Actual Performance
Once the standards are set, the next step is to measure the actual performance of the activities.
This may be done through various techniques such as personal observation, checking the
sample, performance reports, etc. The checking should be done in an exact and reliable
manner so that correct measurement is taken for comparison. Measurement can be done after
the completion of an activity as well as while it is in progress. For example, while assembling
small parts of a bigger machine, the parts can be checked before assembling. This would
ensure the continuous monitoring of the small parts as well as the final machine.
(iii) Comparing the Performances
Performances once measured are then compared with the set standards. Such a comparison
helps in assessing the deviations in the work. Thereby, it guides the managers in taking the
necessary steps so as to improve the performances. These comparisons are easier when they
are in quantitative terms. For example, efficiency in work in terms of cost incurred can be
measured against the standard cost.
(iv) Analysing Deviation
Every organisation faces deviations when comparing the actual performance with the predeveloped standards. Thus, it is important to find the deviations that are in the permissible
range. It is said that deviations in key areas should be attended first. For analysing the
deviations the managers generally use 'Critical Point Control' and 'Management by
Exception'. .
Critical Point Control: An organisation cannot keep a check on all the activities of the
management. Thus, this technique of controlling aims at focussing on only the key result
areas (KRAs) that affect the entire organisation. For example, rise in input cost would be
more important than rise in stationary cost.
Management by Exception: This technique of management is based on the belief that 'an
attempt to control everything results in controlling nothing'. According to this, only the
essential and significant deviations that are beyond the acceptable limit should be controlled.
For example, if there is a 6 per cent rise in labour cost whereas the permissible limit is just 3
per cent, then, this should be immediately brought into the notice of the management. On the
other hand a 2 percent rise in the cost can be ignored.

Once the deviations are recognised, it is necessary to acknowledge the cause for it. There can
be a number of elements causing deviations in work such as infeasible standards, deficiencies
in process, under utilisation of resources, changes in business environment, etc. Thus, it

becomes important for the management to take into regard the causes for the concerned
deviations.
(v) Corrective Measures
When deviations go beyond the admissible limits, there arises a need for the management to
take corrective actions. This is the last step of controlling which aims at correcting the
deficiencies of the organisation so that the errors do not occur again. For example, if the
production target was not met duly, appropriate corrective actions such as training the
workers or updating the machinery for working, etc. can be taken.
Answer needs Correction? Click Here
Q3 :
'Planning is looking ahead and controlling is looking back'. Comment.
Answer :

Planning is looking ahead and controlling is looking back. This statement is partially true.
Planning is a psychological process of 'thinking and deciding in advance' about 'what is to be
done' and 'how it is to be done'. It is a mental activity that includes deciding the goals and
also the actions through which they are to be accomplished. Thus, it is said that planning is
looking ahead as it involves predicting the future. Controlling on the other hand, involves an
assessment of the past performance and evaluating them against the set standards. In this
sense, controlling is said to be a backward looking function.
However, both these statements are only partially true. Though planning is a futuristic
concept but it is based on past actions and experiences. Planning for future cannot take place
without peeping into the past. Similarly, though controlling involves assessment of past
performance, it also aims at improving the future performance by taking the required
corrective actions. Hence, we can say that planning and controlling are backward looking as
well as forward looking functions.
Answer needs Correction? Click Here
Q4 :
Explain the techniques of managerial control.
Answer :

The techniques used for managerial control can be divided into two broad categories namely,
Traditional Techniques and Modern Techniques.
Traditional Techniques

Techniques which are being used by the managers since long back, are known as traditional
techniques. The following are traditional techniques of managerial control.
i. Personal Observation: This technique includes personal observation by the managers to
oversee the work being done. It enables the manager to gather the right information and also
creates a pressure on the workers to perform well as they are being continuously observed by
their supervisor. However, it is a time consuming process and cannot be used where there are
a variety of functions to be overseen.
ii. Statistical Reports: Information in the form various statistical analysis such as averages,
ratios, percentages, etc. can be easily presented in the form of graphs, charts and tables. Such
presentation facilitates easy comparison of the performance with the standards.
iii. Break-Even Analysis: It involves a study of relation between costs, volume and profits.
Break-Even point refers to that quantity of sales where there is neither profit nor loss. It is
determined at the point where total cost incurred is equals the total revenue earned. Through
this technique, the manager can estimate the costs and profits to the organisation at various
levels of quantity and thereby, find the level where profit can be maximised.
iv. Budgetary Control: Budgetary control is a technique of planning the future operations in
the form of budgets. Here, 'budget' refers to a quantitative or qualitative statement which
presents the objectives to be achieved in a specified period of time. These budgets are then
used as standards for measuring the actual performance. It also presents the time-bounded
policies to be used for the attainment of the objectives. It also facilitates management by
exception by focussing on the activities which deviate significantly from the set budgets.
However, to ensure effectiveness of the technique, estimates about the future should be as
accurate as possible. In addition, the budgets should be flexible so as to adapt to the changes
in business environment.
Modern Techniques
Modern techniques as the name suggests are modern and recent in origin. They are based on
the new thinking of the managers and provide refreshing ideas for a better managerial
control. Following are the highlighted modern techniques of controlling.
i. Return on Investment: Return on investment refers to the gains or benefits earned in
relation to the investments done. It is a useful technique in measuring whether the invested
capital is being used effectively and if a reasonable amount of returns are being generated
from these investments. Managers can opt for this technique when comparing the
performances of different departments or divisions or when comparing the present actions in
relation to the previous year performance.
ii. Ratio Analysis: This technique involves calculating various ratios in order to analyse the
financial statements. These ratios are then used as a tool for effective managerial control.
Following are the most commonly used ratios for controlling.
(a) Liquidity Ratio, for determining the short-term solvency of business.

(b) Solvency Ratio, for determining the long-term solvency of a business.


(c) Profitability Ratios, for determining the profitability positions of a business.
(d) Turnover Ratios, for determining the efficiency of activities based on the utilisation of
resources.
iii. Responsibility Accounting: Under this system, different divisions of an organisation are
established as responsibility centres. The head of each centre is responsible for the targets and
duties regarding his centre. The following are some of the responsibility centres that can be
formed.
(a) Cost Centre, is responsible for the costs incurred by the organisation.
(b) Revenue Centre, responsible for the revenue generated from the sales or marketing
activities.
(c) Profit Centre, responsible for the profits generated considering the costs and revenues.
(d) Investment Centre, it takes into account the investment made in the form of assets.
iv. Management Audit: It refers to a systematic approach for analysing and appraising the
overall efficiency of the management of a company. It aims at reviewing the efficiency and
effectiveness of the management in order to identify the deficiencies in the overall
performance. It acts as an important control system by continuously monitoring the working
activities of the managers.
v. PERT and CPM: Programme Evaluation and Review Technique (PERT) and Critical Path
Method (CPM) are techniques that are based on network analysis. It involves dividing the
entire project into various activities and then deciding a time line and cost estimate for each
activity and for the entire project. As these techniques deal with time scheduling and resource
allocation, they enable effective execution of the projects. Such techniques are generally used
in ship-building, construction projects, etc.
vi. Management Information System: MIS is a computer based controlling technique that
provides timely data and information to the managers while aiming at effective decision
making. It processes the massive data of the organisation and generates useful information to
the managers. MIS also ensures cost effectiveness in managing information as it facilitates
collection and dissemination of information at different levels.
The aforementioned traditional and modern techniques can be used by the managers for
effective and efficient controlling of the organisation.
Answer needs Correction? Click Here
Q5 :
'An effort to control everything may end up in controlling nothing'. Explain.

Answer :

The statement, 'an effort to control everything may end up in controlling nothing' is in regard
with the principle of 'Management by Exception'. It stresses on the fact that everything cannot
be effectively controlled. According to this principle, rather than controlling each and every
deviation in performance, an acceptable limit of deviations in various activities should be set
and only those deviations that go beyond the acceptable range should be brought to the notice
of the managers for control. In other words, only the major deviations which are beyond
permissible limit should be acknowledged. For instance, suppose the acceptable range of
increase in the input cost is set at 3 percent. In this case, only a more than 3% increase in the
input cost (say 7%) should be brought to the notice of the managers. On the other hand, a less
than 3% increase (say 1%) should be neglected. Hence, an effort should be there to control
only the major things instead of trying to control everything.
Answer needs Correction? Click Here
Q6 :
Explain the importance of controlling in an organisation. What are the problems faced
by the organisation in implementing an effective control system?
Answer :

Controlling is an important and an indispensable function of management. It aims at


managing the managerial actions by setting the standards and identifying the deviations of
actual performance as against the set standards. It also ensures optimum utilisation of
resources while taking corrective measures for the deviations. The following are the factors
that highlight the importance of controlling.
(i) Achieving Organisational Goals: Controlling aims at accomplishment of the
organisational goals by indicating the deficiencies and the corrective actions which are to be
taken. It helps in moving in the right direction for attaining the set organisational objectives.
(ii) Evaluating the Standards: Controlling helps in judging the accuracy of the standards
adopted by the management. A good controlling system enables the manager to check
whether the set standards are accurate and feasible. It also helps the organisation to review
and revise the standards according to the changing business environment.
(iii) Optimum Utilisation of Resources: A continuous control and monitoring helps in the
efficient and optimum utilisation of resources. As each work is done according to the set
standard, there is less wastage and spoilage of resources.
(iv) Employee Motivation: By exercising effective control, employees get to know well in
advance about what is expected from them and the standards against which their performance
will be assessed. This motivates them to achieve the assigned targets in a better way.

(v) Order and Discipline: Efficient controlling helps in creating an atmosphere of order and
discipline in the organisation. As the employees are aware of the fact that they are being
continuously observed, dishonesty and inefficiency in behaviour is minimised.
(vi) Promoting Coordination: Pre-determined standards provide a basis for better
coordination within various activities. As the departments are made aware of their duties and
tasks, controlling promotes coordination among them. Controlling provides unity of direction
while ensuring that the organisational objectives are met.
Hence, Controlling is an important function that is performed by all the managers. However,
controlling has some limitations. The following points highlight the problems faced by the
organisation when implementing an effective controlling system.
(i) Complication While Setting Standards: It is important to set the standards in quantitative
terms as well as qualitative terms for better controlling. However, controlling becomes less
effective when the standards are defined in qualitative terms. Qualitative standards make the
evaluation of the performance and the comparison of actual work with the standards, a
complicated task. Thus, it might pose a problem in the process of controlling.
(ii) External Factors: Business environment keeps on changing and the organisation have
very little control over such external factors. These factors might create hurdles in effective
controlling. Such factors can be in the form of change in government policies, environmental
changes, competition, etc.
(iii) Resistance from Employees: Controlling can be resisted by the employees if it goes
against their comfort zone and freedom. For instance, if the managers set a defined quantity
for production as a standard and if the workers take it as unrealistic, then they can go on
strike.
(iv) Expensive Process: Effective controlling is a costly affair in terms of time, money and
effort. For example, setting up of CCTVs involves a lot of costs. Thus, it may not be possible
for a small organisation to set up such system. Thereby, the managers should ensure that the
costs incurred in operating such controlling systems do not exceed the benefits derived from
it.
Answer needs Correction? Click Here
Q7 :
Write a short note on budgetary control as a technique of managerial control.
Answer :

Budgetary control is a technique of controlling that involves preparing plans in the form of
budgets. Budget refers to a financial or a quantitative statement that defines the targets to be
achieved and the policies to be followed in a specific period of time. The actual performance
is then compared with the budgetary standards. This comparison helps in identifying the

deviations and thereby, guides in taking appropriate corrective measures. Budget can be
prepared for different divisions of the organisation such as sales budget, production budget,
purchase budget, etc. However, for the budgeting to be effective, future estimates must be
made carefully. Budgeting also acts as a source of motivation for the employees by setting the
standards against which their performance will be assessed. Thus, it encourages them to
achieve the set objectives. In addition, it is also used to facilitate coordination among
different divisions/departments of the organisation. Moreover, proper budgeting ensures that
resources are allocated to different divisions as per their requirements. Thereby, it helps in
optimum utilisation of the resources.
Answer needs Correction? Click Here
Q8 :
Discuss the relationship between planning and controlling.
Answer :

Planning and controlling are closely interrelated functions of management. On one hand,
planning refers to the psychological process of thinking and deciding about what is to be
done and how it is to be done. That is, planning decides the objectives to be achieved and the
course of action to be followed. On the other hand, controlling refers to the process of
managing and evaluating the work done in accordance with the standards and taking
corrective measures, if there are any deficiencies. These standards that form the basis of
controlling are provided by planning. The various objectives and policies as formulated under
planning serve as standards against which the actual performance is evaluated. Controlling
without planning is meaningless. If there are no standards and no objectives, there is nothing
to control. That is, if the managers do not know what the final objective is, they do not have
any standard against which they can judge the current performance and deficiencies.
Similarly, planning without controlling holds no meaning. Once the plans are formulated, it
becomes necessary to monitor and evaluate whether the performance is as per the desired
plans. Controlling is a requisite for measuring whether the plan is being properly
implemented, if there are any deficiencies in the work and if there is a need for taking
corrective actions in order to achieve the planned goals. If there is no controlling, planning
cannot be accomplished. Thereby, planning without controlling is of no use. Thus, it can be
said that both planning and controlling complement each other.
Both the concepts of planning and controlling are interlinked in a way as they are forward
looking as well as backward looking. Though it is often said that planning is looking forward
and controlling is looking backward, however, the statement is only partially true. Though
planning is a futuristic concept dealing with preparing plans and setting standards in advance,
it is also based on past experiences and actions initiated in the controlling function. Thus,
planning besides being forward looking, is backward looking as well. Similarly, although
controlling is based on the past actions and deals with comparing the current actions with the

pre-defined standards, it also focuses on taking the corrective actions to improve the future
performance of the management. Thus, controlling besides looking back, also looks forward.
Hence, while on one hand, planning is a prerequisite for controlling, on the other hand,
controlling is incomplete without planning. Both are inseparable functions that support each
other towards the achievement of the goals of the organisation.
Answer needs Correction? Click Here
Q9 :
Explain how management audit serves as an effective technique of controlling.
Answer :

Management audit refers to the extensive and constructive appraisal of the overall
performance of the management of an organisation. It aims at improving the overall
effectiveness and efficiency of the management. It evaluates all the functions performed by
the managers and helps in identifying the deficiencies in the work performance. The
effectiveness of management audit for controlling can be judged from the following points.
i. Identification of Deficiencies: Management audit helps in recognising the current as well
as probable deficiencies in the performances. Thereby, it helps in taking the necessary
corrective measures.
ii. Improves Efficiency: Through management audit, various activities of the management
can be continuously monitored. Thereby, it helps in improving the overall efficiency of the
management.
iii. Enhances Coordination: It improves the coordination between employees as well as
within the different functions of the organisation as it continuously oversees the work.
iv. Adapting to Environmental Changes: It helps the organisation to adapt to the
environmental changes appropriately. This is done by ensuring that the managerial policies
and strategies are up-to-date.

Chapter 9

Q1 :
What is meant by capital structure?
Answer :

Capital structure refers to the combination of borrowed funds and owners'


fund that a firm uses for financing its fund requirements. Herein, borrowed
funds comprise of loans, public deposits, debentures, etc. and owners' fund
comprise of preference share capital, equity share capital, retained earning
etc. Generally, capital structure is simply referred as the combination of debt
and equity that a firm uses for financing its funds. It is calculated as the ratio
of debt and equity or the proportion of debt in the total capital used by the
firm. Algebraically,

The proportion of the debt and equity used by the firm affects its financial
risk and profitability. While on one hand, debt is a cheaper source of finance
than equity and lowers the overall cost of capital but on the other hand,
higher use of debt, increases the financial risk for the firm. Thus, the decision
regarding the capital structure should be taken with utmost care. Capital
structure is said to be optimal when the proportion of debt and equity used is
such that the earnings per share increases.
Answer needs Correction? Click Here
Q2 :
What is working capital? How is it calculated? Discuss five important
determinants of working capital requirement.
Answer :
Every business needs to take the decision regarding the investment in
current assets i.e. the working capital. Current assets refer to the assets that
are converted into cash or cash equivalents in a short period of time (less
than or equal to one year). There are two broad concepts of working capital
namely, Gross working capital and Net working capital.
Gross working capital (or, simply working capital) refers to the investment
done in the current assets. Net working capital, on the other hand, refers to
the amount of current assets that is in excess of current liabilities. Herein,
current liabilities are those obligatory payments which are due for payment
such as bills payable, outstanding expenses, creditors, etc. Net Working
Capital is calculated as the difference of current assets over current liabilities.
i.e.
NWC = Current Assets - Current Liabilities
The following are five determinants of working capital requirement.

i) Type of Business: Working capital requirement of a firm depends on its


nature of business. An organisation that deals in services or trading will not
require much of working capital. This is because such organisations involve
small operating cycle and there is no processing done. Herein, the raw
materials are the same as the outputs and the sales transaction takes place
immediately. In contrast to this, a manufacturing firm involves large operating
cycle and the raw materials need to be converted into finished goods before
the final sale transaction takes place. Thereby, such firms require large
working capital.
ii) Scale of Operations: Another factor determining the working capital
requirement is the scale of operations in which the firm deals. If a firm
operates on a big scale, the requirement of the working capital increases.
This is because such firms would need to maintain high stock of inventory
and debtors. In contrast to this, if the scale of operation is small, the
requirement of the working capital will be less.
iii) Fluctuations in Business Cycle: Different phases of business cycle alter
the working capital requirements by a firm. During boom period, the market
flourishes and thereby, there is higher sale, higher production, higher stock
and debtors. Thus, during this period the need for working capital increases.
As against this, in a period of depression there is low demand, lesser
production and sale, etc. Thus, the working capital requirement reduces.
iv) Production Cycle: The time period between the conversion of raw
materials into finished goods is referred as production cycle. The span of
production cycle is different for different firms depending on which the
requirement of working capital is determined. If a firm has a longer span of
production cycle, i.e. if there is a long time gap between the receipt of raw
materials and their conversion into final finished goods, then there will be a
high requirement of working capital due to inventories and related expenses.
On the other hand, if the production cycle is short then requirement of
working capital will be low.
v) Growth Prospects: Higher growth and expansion is related to higher
production, more sales, more inputs, etc. Thus, companies with higher growth
prospects require higher amount of working capital and vice versa.
Answer needs Correction? Click Here
Q3 :
Discuss the two objective of Financial Planning.
Answer :

Financial Planning involves designing the blueprint of the financial operations


of a firm. It ensures that just the right amount of funds are available for the
organisational operations at the right time. Thereby, it ensures smooth
functioning. Taking into consideration the growth and performance, through
financial planning, firms tend to forecast what amount of fund would be
required at what time. The following are the two highlighted objectives of
financial planning.
i) Ensure Availability of Funds
Ensuring that the right amount of funds are available at the right time is one
of the main objectives of financial planning. It involves estimating the right
amount of funds that are required for various business operations in the long
term as well for day to day operations. In addition, it also involves estimating
the time at which the funds would be required. Thus, financial planning
ensures that right amount of funds are available at the right time. Financial
planning also points out the probable sources of funds.
ii) Proper Utilisation of Funds
Financial Planning aims at full utilisation of funds. It ensures that both
inadequate funds as well as excess funds are avoided. Inadequate funds
hinders the smooth operations and the firm is unable to carry its
commitments. On the other hand, excess funds add to the cost of business
and encourage unnecessary wasteful expenditure. Thus, financial planning
ensures that the funds are properly and optimally utilised.
Answer needs Correction? Click Here
Q4 :
''Capital structure decision is essentially optimisation of risk-return
relationship''. Comment.
Answer :
Capital Structure refers to the combination of different financial sources used
by a company for raising funds. The sources of raising funds can be classified
on the basis of ownership into two categories as borrowed funds and owners'
fund. Borrowed funds are in the form of loans, debentures, borrowings from
banks, public deposits, etc. On the other hand, owners' funds are in the form
of reserves, preference share capital, equity share capital, retained earnings,
etc. Thus, capital structure refers to the combination of borrowed funds and
owners' fund. For simplicity, all borrowed funds are referred as debt and all
owners' funds are referred as equity. Thus, capital structure refers to the
combination of debt and equity to be used by the company. The capital

structure used by the company depends on the risks and returns of the
various alternative sources.
Both debt and equity involve their respective risk and profitability
considerations. While on one hand, debt is a cheaper source of finance but
involves greater risk, on the other hand, although equity is comparatively
expensive, they are relatively safe.
The cost of debt is less because it involves low risk for lenders as they earn
an assured amount of return. Thereby, they require a low rate of return which
lowers the costs to the firm. In addition to this, the interest on debt is
deductible from the taxable income (i.e. interest that is to be paid to the debt
security holders is deducted from the total income before paying the tax).
Thus, higher return can be achieved through debt at a lower cost. In contrast,
raising funds through equity is expensive as it involves certain floatation cost
as well. Also, the dividends are paid to the share holders out of after tax
profits.
Though debt is cheaper, higher debt raises the financial risk. This is due to
the fact that debt involves obligatory payments to the lenders. Any default in
payment of the interest can lead to the liquidation of the firm. As against this,
there is no such compulsion in case of dividend payment to shareholders.
Thus, high debt is related to high risk.
Another factor that affects the choice of capital structure is the return offered
by various sources. The return offered by each source determines the value
of earning per share. A high use of debt increases the earning per share of a
company (this situation is called Trading on Equity). This is because as debt
increases the difference between Return on Investment and the cost of debt
increases and so does the EPS. Thus, there is a high return on debt. However,
even though higher debt leads to higher returns but it also increases the risk
to the company.
Therefore, the decision regarding the capital structure should be taken very
carefully, taking into consideration the return and risk involved.
Answer needs Correction? Click Here
Q5 :
What is financial risk? Why does it arise?
Answer :
Financial risk refers to a situation when a company is not able to meet its
fixed financial charges such as interest payment, preference dividend and

repayment obligations. In other words, it refers to the probability that the


company would not be able to meet its fixed financial obligations. It arises
when the proportion of debt in the capital structure increases. This is because
it is obligatory for the company to pay the interest charges on debt along
with the principle amount. Thus, higher the debt, higher will be its payment
obligations and thereby higher would be the chances of default on payment.
Hence, higher use of debt leads to higher financial risk for the company.
Answer needs Correction? Click Here
Q6 :
''A capital budgeting decision is capable of changing the financial
fortunes of a business''. Do you agree? Why or why not?
Answer :
Yes, capital budgeting decision is a very essential decision which needs to be
taken carefully. It has the capability of changing the financial fortunes of a
business. Capital budgeting decision refers to the decisions regarding the
allocation of fixed capital to different projects. Such decisions involve
investment decisions regarding attainment of new assets, expansion,
modernisation and replacement. Such long term investments include
purchasing plant and machinery, furniture, land, building, etc. and also
expenditure as on launch of a new product, modernisation and advertising,
etc. They have long term implications on the business and are irrevocable
except at a huge cost. They affect a business' long term growth, profitability
and risk.
The following are the factors that highlight the importance of capital
budgeting decisions.
i) Long Term Implications: Investment on capital assets (long term assets)
yield return in the future. Thereby, they affect the future prospects of a
company. A company's long term growth prospects depend on the capital
budgeting decisions taken by it.
ii) Huge Amount of Funds: Investing in fixed capital involves a large
amount of funds. This makes the capital budgeting decisions all the more
important as huge amount of funds remain blocked for a longer period of
time. These decisions once made are difficult to change. Thus, capital
budgeting decisions need to be taken carefully after a detailed study of the
total requirement of funds and the sources from which they are to be raised.
iii) High Risk: Fixed assets involve huge amount of money and thereby,
involve huge risk as well. Such decisions are risky as they have an impact on

the long term existence of the company. For example, decision about the
purchase of new machinery involves a risk in terms of whether the return
from the machinery would be greater than the cost incurred on it.
iv) Irreversible Decisions: These decisions once made are irrevocable.
Reversing a capital budgeting decision involves huge cost. This is because
once huge investment is made on a project, withdrawing it would mean huge
losses.
Answer needs Correction? Click Here
Q7 :
Define a 'current asset'. Give four examples of such assets.
Answer :
Current asset of a firm refers to those assets which can be converted into
cash or cash equivalents in a short period of time, i.e. less than one year.
Such assets are used to facilitate the day to day business operations. As they
can be easily converted into cash or cash equivalents, these assets provide
liquidity to the company. Firms acquire such assets to meet its various
payment obligations. However, such assets provide very little return and are
thereby, less profitable. Current assets can be financed through short-term as
well as long term sources.
Some of the examples of current assets are short term investment, debtors,
stocks and cash equivalents.
Answer needs Correction? Click Here
Q8 :
Explain the factors affecting the dividend decision.
Answer :
Dividend decision of a company deals with what portion of the profits is to be
distributed as dividends between the shareholders and what portion is to be
kept as retained earnings. The following are the factors that affect the
dividend decision.
i) Amount of Earning: A firm pays dividends out of its current and the past
earnings. This implies that earnings play a key role in the dividend decision. A
company having higher earnings will be in a position to pay a higher amount

of dividend to its shareholders. In contrast to this, a company having low or


limited earnings would distribute low dividends.
ii) Stable Earnings: When a company has a stable and a smooth earning,
they are in a position to distribute higher dividend as compared to the
companies who have an unstable earning. In other words, a company having
consistent and stable earnings can distribute higher amount of dividends.
iii) Stable Dividends: Companies generally follow the practice of stabilising
their dividends. They try to avoid frequent fluctuations in dividend per share
and opt for increasing (or decreasing) the value only when there is a
consistent rise (or fall) in the earnings of the company.
iv) Growth Prospects: Companies aiming for a higher growth level or
expansion of operations retain a higher portion of the earnings with itself for
re-investment. Thus, dividend of such a company is smaller as compared to
the companies with lower growth opportunities.
v) Cash Flow Position: Dividend payments require cash outflow. If a
company is low on cash then the dividend will be lower as compared to the
company which has more liquidity. Even if a company has higher profits, it
will not be able to distribute high dividends if it does not have enough cash.
vi) Preference of the Shareholders: A company must keep in mind the
preferences of the share holders while distributing the dividends. For
instance, if the share holders prefer at least a certain amount of dividend,
then the company is likely to declare the same.
vii) Taxation Policy: Taxation policy plays an important role in deciding the
dividends. If the taxation policy is such that a high rate of tax is levied on
dividend distribution, then the companies are likely to distribute lower
dividends. On the other, it might prefer to distribute higher dividends if the
tax rate is low.
viii) Stock Market Reactions: The amount of dividend that a company
distributes affects its stock market prices. An increase in dividend by a
company is viewed as a good sign by the investors and the stock price of the
company goes up. On the other hand, a fall in the dividends adversely affects
the stock prices. Thus, while taking the dividend decision, a company must
consider the probable stock market reactions.
ix) Contractual Constraints: Sometimes, while giving out loans to a
company, the lender may impose some restrictions in the form of agreement.
These restrictions may be related to the dividend paid in the future. In such
cases, the company has to keep such agreements in mind when distributing
the dividends.

x) Access to Capital Market: The companies that have a greater access to


the capital market tend to pay higher dividends. This is because they can rely
less on retained earnings and more on other sources due to the market
access. The smaller companies who have lower access to capital markets
tend to pay lower dividends.
xi) Legal Constraints: Companies have to adhere to the rules and policies
laid out by the Companies Act. Thus, any company needs to take care of such
restrictions and policies before declaring the dividends.
Answer needs Correction? Click Here
Q9 :
Financial management is based on three broad financial decisions.
What are these?
Answer :
Financial management refers to the efficient acquisition, allocation and usage
of funds of the company. It deals in three main dimensions of financial
decisions namely, Investment decisions, Financial decisions and Dividend
decisions.
Investment Decisions
Investment decisions refer to the decisions regarding where to invest so as to
earn the highest possible returns on investment. Investment decisions can be
taken for both long term as well as short term.
Long term investment decisions also known as Capital Budgeting decisions
affect a business' long term earning capacity and profitability. For example,
investment in a new machine, purchase of a new building, etc. are long term
investment decisions.
Short term investment decisions also known as working capital decisions
affect a business' day to day working operations. For example, decisions
regarding cash or bill receivables are short term investment decisions.
Financial Decisions
Such decisions involve identifying various sources of funds and deciding the
best combination for raising the funds. The main sources for raising funds are
shareholders' funds (referred as equity) and borrowed funds (referred as
debt). Based on the cost involved, risk and profitability a company must
judiciously decide the combination of debt and equity to be used. For
example, while debt is considered to be the cheapest source of finance,

higher debt increases the financial risk. Financial decisions taken by a


company affects its overall cost of capital and the financial risk.
Dividend Decisions
The decision involves the decision regarding the distribution of profit or
surplus of the company. A company can distribute its profit to the equity
share holders in the form of dividends or retain it with itself. Under dividend
decision, a company decides what proportion of the surplus to distribute as
dividends and what proportion to keep as retained earnings. It is aimed at
maximising the shareholders' wealth while keeping in view the requirement
of retained earnings that are needed for re-investment.
Answer needs Correction? Click Here
Q10 :
What are the main objectives of financial management? Briefly
explain
Answer :
The paramount objective of the financial management is maximising the
shareholders' wealth. That is, the basic objective of financial management for
a company is to opt for those financial decisions that prove gainful from the
point of view of the shareholders. The share holders are said to gain when the
market value of their shares rise. The market value of shares increase when
the benefits from a financial decision exceed the cost involved in taking
them. In other words, a financial decision raises the market value of share if it
results in some value addition. Thus, financial decisions should be taken such
that some value addition takes place and ultimately the price of the equity
share increases. When a financial decision is able to fulfil the primary
objective of wealth maximisation, other objectives such as proper utilisation
of funds, maintenance of liquidity etc. are automatically fulfilled.
Answer needs Correction? Click Here
Q11 :
How does working capital affect both the liquidity as well as
profitability of a business?
Answer :
Working capital of a business refers to the excess of current assets (such as
cash in hand, debtors, stock, etc.) over current liabilities. Working capital

affects both the liquidity as well as profitability of a business. As the amount


of working capital increases, the liquidity of the business increases. However,
since current assets offer low return, with the increase in working capital the
profitability of the business falls. For example, an increase in the inventory of
the business increases its liquidity but since the stock is kept idle, the
profitability falls. On the other hand, low working capital, hinders the day to
day operations of the business. Thus, the working capital should be such that
a balance is maintained between the profitability and liquidity.
Answer needs Correction? Click Here
Q12 :
Higher dividend per share is associated with
(a) high earnings, high cash flows, unstable earnings and higher
growth opportunities
(b) high earnings, high cash flows, stable earnings and high growth
opportunities
(c) high earnings, high cash flows, stable earnings and lower growth
opportunities
(d) high earnings, low cash flows, stable earnings and lower growth
opportunities
Answer :
If a company gives higher dividend per share then it gets associated with
high amount of earnings as only if they will earn higher, they will be able to
give higher dividends; higher cash flow as the payment of dividend involves
cash outflow; stable earnings as stable earnings means that the company is
confident of its future earning potentials; and lower growth opportunities
because it requires less requirement of retained earnings and their retained
earnings while lowering the amount of dividends paid.
Answer needs Correction? Click Here
Q13 :
A fixed asset should be financed through
(a) a long term liability
(b) a short term liability
(c) a mix of long and short term liabilities

Answer :
Fixed assets are those assets which are invested in a company for a longer
time period, generally more than one year. As these assets have long term
implication on the business in terms of growth and profitability, they should
be financed through long term liabilities such as long term loans, preference
shares, retained earnings, etc.
Answer needs Correction? Click Here
Q14 :
Current assets of a business firm should be financed through
(a) current liability only
(b) long-term liability only
(c) both types (i.e. Long and short liabilities)
Answer :
Current assets are those assets which get converted in cash or cash
equivalents within a short span of time and provide liquidity to a business.
For financing the current assets of a business, both types of liabilities (short
and long) can be used.
Answer needs Correction? Click Here
Long answers : Solutions of Questions on Page Number : 266
Q1 :
Explain the term ''Trading on Equity''. Why, when and how it can be
used by a company?
Answer :
Trading on equity refers to a practice of raising the proportion of debt in the
capital structure such that the earnings per share increases. A company
resorts to Trading on Equity when the rate of return on investment is greater
than the rate of interest on the borrowed fund. That is, the company resorts
to Trading on Equity in situation of favourable financial leverage. As the
difference between the return on investment and the rate of interest on debt
increases, the earnings per share increase.
The use of Trading on Equity is explained in detail with the help of the
following example.

Suppose there are two situations for a company. In situation I it raises a fund
of Rs 5,00,000 through equity capital and in situation II, it raises the same
amount through two sources- Rs 2,00,000 through equity capital and the
remaining Rs3,00,000 through borrowings.
Also suppose the tax rate is 30% and the interest on borrowings is 10%. The
earnings per share (EPS) in the two situations is calculated as follows.

Situation I

Situation II

1,00,000

1,00,000

1,00,000

30,000
70,000

Tax

30,000

21,000

Earnings After Tax (EAT)

70,000

79,000

No. Of equity shares

50,000

20,000

Earnings before interest and tax


(EBIT)
Interest
Earnings Before Tax (EBT)

Clearly, in the second situation the EPS is greater than in the first situation. In
the second situation the company takes advantage of the Trading on Equity
and raises the EPS. Here, the return on investment calculated
as
is 20% while the interest on the
borrowings is 10%. Thus, the Trading on Equity is profitable.
However, it should be noted that Trading on Equity is profitable and should be
used only when the return on investment is greater than the interest on
borrowed funds. In case the return on investment is less than the rate of
interest to be paid, the Trading on Equity should be avoided.
Suppose instead of Rs 1,00,000 the company earns just Rs 25,000. In such a
case the EPS are calculated as follows.

Situation I

Situation II

Earnings before interest and tax


(EBIT)
Interest
Earnings Before Tax (EBT)

40,000

40,000

25,000

10,000
10,000

Tax

30,000

3,000

Earnings After Tax (EAT)

70,000

7,000

No. Of equity shares

50,000

20,000

Clearly in this case, the EPS in Situation II falls. Here the return on investment
is only 8%
while the interest on the
borrowings is 10%. Thus, in this situation the Trading on Equity is not
favourable and should be discouraged.
Hence, it can be said that a firm can use Trading on Equity if it is earning high
profits and can increase the EPS by raising more funds through borrowings.

Chapter 10

Q1 :
What are the functions of a financial market?
Answer :
A financial market refers to the market where the creation and exchange of
financial assets such as shares and debentures takes place. The following are
the functions of a financial market.
i) Transfer of Savings and Alternatives for Investment
A financial market acts a link between the savers and the investors. It
provides a platform for the transfer of savings from the households to the
investors. It also provides savers with various alternatives for investment and
thereby, directs the funds to the most productive investment.
ii) Establishes the Price
Similar to a commodity, the price of a financial asset is established through
the forces of demand and supply for funds. Financial market provides a
platform for the interaction of the demand of the funds (represented by the
business firms) and the supply of funds (represented by the households).
Thereby, it helps in determining the price of the asset being traded.

iii) Facilitates Liquidity


An asset or a security can be easily purchased and sold in a financial market.
This renders liquidity to the assets. That is, through trading in the financial
market assets can be easily converted into cash or cash equivalents.
iv) Reduced Cost of Transaction
By rendering information regarding the securities being traded, their price,
availability, etc., a financial market helps in reducing the cost of transaction
in terms of effort, money and time.
Answer needs Correction? Click Here
Q2 :
Explain the various Money Market Instruments.
Answer :
Money Market refers to the market where short term funds are traded.
Herein, short term funds are in the form of monetary assets having a maturity
period of maximum one year. The following are some of the common money
market instruments.
(i) Treasury Bil (T-Bills)
Treasury Bill refers to a promissory note used for short term borrowing by the
government. They are the most commonly used money market instrument.
They are auctioned and issued by the Reserve Bank of India on behalf of the
Central Government. T-bills are available for a minimum of Rs 25,000 and in
multiples thereof. Generally, three types of treasury bills are issued 91-days,
182-days and 364-days. T-Bills are issued at a discount and redeemed at par.
That is, they are issued at a price lower than their face value and at the time
of redemption, the investor gets the amount equal to the face value. The
difference between the value at which they are issued and the redemption
value is the interest received on them. For example, if an investor purchases
a 182-days treasury bill with a face value of Rs 56,000 for Rs 50,000. At the
time of maturity, the investor will receive Rs 56,000. Thus, the difference of
Rs 6,000 (56,000 - 50,000) is the interest receivable on the bill. T-Bills are
also called Zero-Coupon Bonds. T-bills are highly liquid bonds. Moreover, as
they are issued by the RBI, they have negligible risk and offer assured return.
(ii) Call Money
Call money is an instrument used for interbank transactions. Through call
money, the banks borrow from each other to meet any shortage of funds

required to maintain CRR. That is, any bank in shortage of funds borrows from
other bank having surplus funds. Call money have a very short maturity
period ranging from one day to fifteen days. Interest paid on such loans is
known as call rate. Call rate is highly volatile and varies from day to day.
There exists a negative relationship between call rate and other money
market instruments such as Commercial Papers and Certificate of Deposits.
That is, as the call rate rise, other instruments of money market become
cheaper and their demand increases.
(iii) Commercial Paper (CPs)
Commercial paper is an unsecured short term money market instrument. It is
a negotiable and transferable promissory note with a maturity period ranging
from a minimum of 15-days to a maximum of one year. They were introduced
in India in 1990. CPs are mainly issued by large and creditworthy companies
to raise short-term funds. Large companies view Commercial Papers as an
alternative to bank borrowings and borrowings through capital market. The
rate of interest payable on Commercial Papers is lower than the market rates.
Generally, companies use Commercial Papers for bridge financing. That is, to
raise the funds required to meet the floatation cost incurred on long term
borrowings in the capital market. For example, if a company wishes to raise
finance from the capital market to purchase land. For this, it will have to incur
floatation cost such as cost related to brokerage, commission, advertising,
etc. To finance such floatation costs the company can issue Commercial
Paper.
(iv) Certificate of Deposit (CDs)
Certificate of Deposits are time deposits which are negotiable and unsecured
in nature. They are bearer instruments for a short and specified time period
ranging from one month to more than five years. CDs are a secured form of
investment, which are issued to individuals, corporations and companies by
the commercial banks and development financial institutions. Herein, higher
interests are offered for higher deposits. They are issued to meet the demand
for credit in times of tight liquidity position. For example, when a person buys
a CD by depositing a specific amount, he receives a certificate wherein the
term of deposit, the interest rate applicable and the date of maturity is
written. On the date of maturity, the individual gets entitled to receive the
principle amount and the earned interest on it.
(v) Commercial Bill
Commercial bill also known as bank bill or bill of exchange refers to the
instrument used to finance the working capital requirements of a firm. It is a
short term negotiable instrument. Companies use Commercial Bills to finance

their credit sales. For example, when an individual makes credit sales, the
buyer becomes liable to make the payment on a specified future date.
Herein, the seller draws a bill of exchange and gives it to the buyer
mentioning a specific maturity period. Once the bill is accepted by the buyer
it becomes a marketable instrument which can be discounted with a bank.
For instance, if the seller requires funds before the maturity period, he can
discount the bill with a commercial bank.
Answer needs Correction? Click Here
Q3 :
''Money Market is essentially a Market for short term funds''.
Discuss.
Answer :
Money market refers to the market for trading of short term securities and
funds. Securities traded in the money market have a very short maturity
period ranging from one day to one year. Such assets act as a close
substitute for cash or money. Due to their short maturity period they are also
known as 'Near Money instruments'. Money market instruments act as an
important source of finance for working capital requirements. They enjoy a
high degree of liquidity. DFHI discounts money market securities and offers a
ready market for them. In addition, securities traded in the money market are
safe and secure as the transactions are made in those instruments that are
issued by financial institutions and those companies that are financially
strong. Common instruments traded in the money market are treasury bills,
commercial paper, call money, certificate of deposit, etc.
Answer needs Correction? Click Here
Q4 :
What are the methods of floatation in Primary Market?
Answer :
The following are the various methods through which floating of new issues
can be done.
(i) Offer through Prospectus
The most commonly used method for raising funds in primary market is offer
through prospectus. It involves inviting the subscriptions from public by issue
of prospectus. A prospectus is published as advertisements in newspapers,

magazines, etc. It provides such information as the purpose for which the
fund is being raised, company's background and future prospects, its past
financial performance, etc. Such information helps the public and the
investors to know about the company as well as the potential risk and the
earnings involved. Such issues need to be listed on one of the stock
exchanges and should be in accordance with the guidelines and rules listed
under the Companies Act and SEBI disclosure.
(ii) Offer through Sale
As against offer through prospectus, under the offer through sale method, the
company does not issue securities directly to the public rather they are
issued through intermediaries such as brokers, issuing houses, etc. That is,
under offer through sale, securities are issued in two steps, first the company
sells its securities to the intermediaries at the face value and later the
intermediaries resell the securities to the investing public at a higher price
than the face value to earn profit.
(iii) Private Placement
Under this method, the securities are sold only to some selected individuals
and big institutional investors rather than to the public. The companies either
allot the securities themselves or they sell the securities to intermediaries
who in turn sell them to selected clients. This method saves the company
from various mandatory or non-mandatory expenses such as cost of manager
fees, commission, underwriter fees, etc. Thus, the companies which cannot
afford the huge expenses related to public issue often go for private
placement.
(iv) Rights Issue
Under the Companies Act 1956, it is the right of the existing share holders of
a company to subscribe to the new shares issued by it. The existing share
holders are offered subscription of new shares of the company in proportion
to the number of shares possessed by them.
(v) e-IPOs
It is system of issuing securities through online system. If a company decides
to offer its securities through an online system it is required to gets into an
agreement with the stock exchange. This is called Initial Public Offer (IPO).
Company appoints brokers for accepting applications and placing orders. A
company can apply to get listed in any stock market except from the one
through which it has already offered securities. Herein, the lead manager
looks upon the various activities and coordinates them.
Answer needs Correction? Click Here

Q5 :
What is a Treasury Bill?
Answer :
Treasury Bill is a short term promissory note issued by the Reserve Bank of
India on behalf of the Central Government of India. They are issued to fulfil
the short-term fund requirements of the Government of India. Maturity period
of Treasury Bills ranges from 14 days to 364 days. Generally, these bills are
brought by commercial banks, LIC, UTI, non-baking financial companies, etc.
They are also called Zero-Coupon Bonds. Treasury bills are highly liquid
instruments because of the fact that the RBI is always ready to purchase
these bills. Moreover, they are also considered to be the safest instrument as
they are issued by the RBI. They are available for a minimum amount of Rs
25,000 and in multiples thereof. Treasury Bills are issued at a discount i.e.
they are issued at a price which is lower than the face value and are
redeemed at par. Herein, the discount (the difference between the price of
issue and the redemption value) is the interest received at the time of
redemption.
Answer needs Correction? Click Here
Q6 :
Explain the recent Capital Market reforms in India.
Answer :
A capital market refers to the market that deals in the trading of medium and
long-term securities. That is, it deals in those securities that have a maturity
period of greater than or equal to one year. Capital market comprises of
instruments such as equity and preference shares, debentures, bonds,
mutual funds, public deposits, etc. A capital market can be divided in two
parts namely, Primary Market and Secondary Market. Primary market deals
with issue of new securities. Issue of new securities in the primary market
directs funds towards those entrepreneurs who either want to start a new
enterprise or wish to expand the existing one. Secondary market, on the
other hand, deals in the sale and purchase of the existing securities. That is,
it deals in the trading of those securities that were initially issued in the
primary market.
The history of capital market in the form of stock exchange dates back to the
eighteenth century. The Government of India introduced the Companies Act
in 1850 with the aim of generating investor interest in corporate securities.

The first stock exchange was set up in India in the year 1875 as 'The Native
Share and Stock Brokers Association' in Bombay. Later it was renamed as
'Bombay Stock Exchange' (BSE). In the subsequent years stock exchanges
were developed in Ahemdabad, Calcutta and Madras.
In 1990s, the Indian secondary market only consisted of regional stock
exchanges wherein, first being the BSE. However, after the reforms of 1991,
the Indian Stock Market acquired a three-tier system. This consisted of
Regional Stock Exchanges, National Stock Exchange and Over the Counter
Exchange of India (OTCEI).
Regional Stock Exchange
The first Regional Stock Exchange was developed in Ahemdabad as
Ahmedabad Stock Exchange (ASE) in 1894. Similarly, in 1908, Calcutta Stock
Exchange (CSE) was established. Subsequently in the later years other
regional stock exchanges were established in Calcutta, Madras, Ahemdabad,
Delhi, Hyderabad and Indore. Recently, regional stock exchanges were
developed in Coimbatore as Coimbatore Stock Exchange and in Meerut as
Meerut Stock Exchange. Currently, there are 22 regional stock exchanges in
India.
National Stock Exchange
The NSE is the latest technology driven stock exchange which was recognised
in 1993. It started its operations in 1994 with trading in money market
securities. Later, it also expanded its trading operations in capital market
segment. NSE was set up in order to establish a nationwide platform for
trading in all types of securities. It ensured development of fair and efficient
securities market. Within the span of its existence, NSE has transformed the
Indian capital market and has been able to take the stock market to the
investor's door step. It has provided a wide screen-based automated trading
system across the nation ensuring equal access to all the investors.
Over the Counter Exchange of India (OTCEI)
OTCEI is a company which was set up in 1990 under the Companies Act,1956
but later was recognised as a stock exchange under the Securities Contracts
Regulation Act, 1956. It commenced its operations in trading in 1992 and is
modelled along the lines of NASDAQ, the OTC exchange in USA. It aims at
providing the small companies an easy access to the capital market. OTCEI
provides a screen based nationwide trading system, that acts as a place
where buyers meet the sellers and negotiate for an acceptable terms of
trade. Herein, dealers can trade both in new issue of securities as well as
secondary market. It is a single window exchange which provides a
convenient, transparent and efficient avenue for capital market investment.

Answer needs Correction? Click Here


Q7 :
Distinguish between Capital Market and Money Market.
Answer :
The following points highlight the difference between Capital Market and
Money Market.

Basis of Difference

Capital Market

Money Market

Time Span of Securities

Capital Market mainly deals in the


trading of medium and long-term
securities wherein, the maturity period
is more than one year.

Money Market deals in the trading


short-term securities wherein, the
maturity period can vary from one
to a maximum of one year.

Liquidity

Capital market securities are liquid in


nature as they are tradable on stock
exchanges, but are less liquid in
comparison to the money market
securities.
Expected returns are higher due to the
possibility of capital gains in long-term
and regular dividends or bonus.

The securities traded are highly liq


in nature. DFHI discounts money
market securities and offers a read
market for them.

Instruments

Instruments traded in capital market


comprise of equity shares, preference
shares, debentures, bonds and other
long term securities.

Instruments traded in money mark


comprise of treasury bills, commer
bills, certificate of deposits and oth
short-term securities.

Risk

Money market securities are less r


Capital market securities involve
greater risk in terms of repayment of the due to short time period and sound
financial position of the issuers.
principal amount.

Returns Expected

Expected returns are lower due to


shorter duration.

Answer needs Correction? Click Here


Q8 :
Explain the objectives and functions of the SEBI.
Answer :
The Securities and Exchange Board of India was established in 1988 in order
to encourage an orderly and healthy growth of the securities market. SEBI

was set with an overall objective of investor protection and to promote the
development and regulation of the functions of the securities market. The
following are the listed objectives.
(i) Regulation: The main objective of SEBI was to regulate the functioning of
the stock exchange and the securities market. It aims at providing a place
where the issuers of securities (i.e. companies) can raise funds in an easy
and confident manner.
(ii) Protection: SEBI educates the investors by providing them valuable
information regarding various securities and companies. It provides them
with the guidelines related to efficient investment. It provides them adequate
and reliable information about the companies and thereby, helps them in
taking wise and informed investment decisions.
(iii) Prevention: To combat the malpractice in trading of securities was the
basic reason for the establishment of SEBI. Malpractice such as insider
trading, violation of rules and regulations, non-adherence to Companies Act,
etc. erodes the confidence of investors. SEBI aims at checking these
malpractice by creating a balance between the self regulation of a business
and the legal statutory regulations.
(iv) Code of Conduct: Through efficient regulation, SEBI aims at developing
a code of conduct for fair trade practices by intermediaries such as brokers,
merchant bankers, underwriters, etc. This helps in making them competitive
and professional.
To attain the aforementioned objectives, SEBI perform 3 main functions
namely, Regulatory, Development and Protective functions. The following are
the functions performed by SEBI.
(i) Regulatory Functions
Registration: One of the regulatory functions performed by SEBI is the
registration of the brokers, sub-brokers, agents and other players in the
market. Registration of collective mutual schemes and Mutual Funds is also
done by SEBI.
Regulating the Work: SEBI regulates the working of the stock brokers,
underwriters, merchant bankers and other market intermediaries. It frames
rules and regulations for the working of the intermediaries. SEBI also
regulates the takeover bids by the companies. It conducts regular enquires
and audits of stock exchange and intermediaries.
Regulation by Legislation: SEBI performs and exercise various other
powers which are delegated by the Government of India under the Securities

Contracts (Regulation) Act, 1956. Besides, it levies fee or other charges for
carrying out the purposes of the Act.
(ii) Development Functions
Training: SEBI promotes the training and development of the
intermediaries of the securities market in order to promote healthy growth of
the securities market.
Research: By conducting research in the required and important areas of
the securities market, SEBI publishes useful information. This helps the
investors and other market players to make wise investment decisions.
Flexible Approach: SEBI has adopted a flexible and adaptive approach
such permitting internet trading, IPOs, etc. Such measures promote the
development of capital market.
(iii) Protective Functions
Prohibition: SEBI prohibits fraudulent and unfair trade practices. It
prevents the spreading of misleading and manipulative statements which are
likely to affect the working of the securities market. SEBI educates the
investors by providing them valuable information regarding various securities
and companies so as to enable them to make wise investment decisions.
Checks on Insider Trading: Insider trading refers to a situation where an
individual connected with the company leaks out crucial information
regarding the company. Such information may adversely affect its share
prices. SEBI keeps a strict check on such insider trading.
Promotion and Protection: SEBI encourage fair trade practices and
promotes a code of conduct for the intermediaries. It undertakes step for
investor protection and education. It also checks the manipulation of price of
securities.
Answer needs Correction? Click Here
Q9 :
What are the functions of a Stock Exchange?
Answer :
Stock Exchange refers to a market where buying and selling of the existing
securities take place. The following are the main functions of a stock
exchange.

(i) Provides Liquidity and Marketability: Stock exchange provides a ready


platform for trading of existing securities. In other words, it provides a
continuous market for the sale and purchase of securities. Through stock
exchange, securities can be easily converted into cash whenever required. In
addition, long-term securities can be converted to medium-term and shortterm through stock exchange.
(ii) Determination of Prices: A stock exchange helps in establishing the
price of the monetary assets that are traded in that market. It provides a
platform for interaction for buyers and sellers of securities and thereby, helps
in the determination of prices of the securities through the forces of demand
and supply.
(iii) Fair and Safe Market: As stock exchange is a legal and well regulated
market. It trades within the defined and the existing legal framework.
Thereby, it ensures safety and fairness in transactions.
(iv) Facilitates Economic Growth: In a Stock Exchange the securities are
continuously brought and sold. This continuous process of disinvestment and
reinvestment helps in channelising the savings and the investments to the
most productive use. This, enhances capital formation and economic growth.
(v) Spreading Equity Cult: Through regulation of the issues and better
trading practices, a stock exchange helps in educating the people about
investment. It promotes and encourages the people to invest in ownership
securities.
(vi) Acts as an Economic Barometer: Through changes in the share prices,
a stock exchange indicates the changes in economic conditions. For instance,
a boom (or recession) is reflected in the rise (or fall) in the share prices.
(vii) Scope for Speculation: It is generally believed that certain degree of
speculation is necessary for better liquidity and to maintain demand and
supply of securities. Stock exchange provides a reasonable and controlled
scope of speculation within the provisions of law.
Answer needs Correction? Click Here
Q10 :
Explain the various segments of the NSE.
Answer :
The National Stock Exchange is the technology driven stock exchange which
was incorporated in 1992. It was recognised as a stock exchange in 1993 and
started operations in the year 1994. NSE provides trading in two main

segments namely, Whole Sale Debt Market Segment and Capital Market
Segment.
(i) Whole Sale Debt Market Segment
This segment provides a platform for trading in fixed income securities such
as state development loans, bonds issued by public sector undertaking,
corporate debentures, commercial paper, mutual funds, central government
securities, zero coupon bonds, treasury bills, etc. NSE started operations in
Whole Sale Debt Market in June 1994. It is the first fully screen based system
for trading in debt market. That is, it is the first computer based trading
system. Trading in the debt market involves two parties- trading members
(which are the recognised brokers of NSE) and the participants (i.e. the
buyers and sellers of securities). The transactions among the participants are
settled through members. For instance, the members place an order for the
seller of a security which is then suitably matched by another member for
buyer of a security wishing to purchase that security. An order remains in the
system until it is suitably matched. This segment of NSE is also known as
NEAT (National Exchange for Automated trading).
(ii) Capital Market Segment
Under this segment, NSE deals with trading in equity shares, preference
shares, debentures, exchange traded funds as well as retail Government
securities. It provides an efficient and transparent platform for a fair trading
system. The capital market segment commenced its working in November
1995. The trading system of NSE Capital Market segment is also known as
the National Exchange for Automated Trading - Capital Market (NEAT- CM).
The trading operations of the Capital Market segment remain the same as in
the Whole Sale Debt market system.
Answer needs Correction? Click Here
Q11 :
What are the objectives of the SEBI?
Answer :
Securities and Exchange Board of India (SEBI) was established for promoting
an orderly and healthy growth of the securities market in India. The following
points highlight the overall objectives of SEBI.
(i) Regulation: The basic objective of SEBI is to regulate the functioning of
stock exchange and the securities market. It aims at providing a place where

the issuers of securities (i.e. companies) can raise funds in an easy and
confident manner.
(ii) Protection: SEBI works on educating the investors and provide guidelines
related to investment. It provides them adequate and reliable information
about the companies and thereby, helps them in taking wise and informed
investment decisions.
(iii) Prevention: To combat the malpractice in trading of securities was the
basic reason for the establishment of SEBI. Malpractice such as insider
trading, violation of rules and regulations, non-adherence to Companies Act,
etc. erodes the confidence of investors. SEBI aims at checking these
malpractice by creating a balance between the self regulation of a business
and the legal statutory regulations.
(iv) Code of Conduct: Through regulation, SEBI develops a code of conduct
for the fair trade practices by the intermediaries such as brokers, merchant
bankers, underwriters, etc. SEBI controls the activities of these intermediaries
and provides them a professional and competitive environment.
Answer needs Correction? Click Here
Q12 :
State the objectives of the NSE.
Answer :
National Stock Exchange of India was incorporated in the year1992. It was
recognised as Stock Exchange in 1993 and started operations in 1994. It was
established by leading banks, financial institutions, insurance companies and
financial intermediaries. NSE was established with the following objectives.
(i) NSE aimed at setting up a single nationwide trading system for providing
the trading facility in all types of securities. Such a system increases the
confidence of the investors.
(ii) It ensured that all the investors over the country get an easy and equal
access through an appropriate communication network. It increases the
liquidity of the securities. Under the system of regional stock exchange the
number of people involved in the transaction was limited. As against this,
NSE incorporates transactions from investors from the entire country and
thereby, increases the liquidity of the securities.
(iii) By using an electronic trading system, NSE aims at providing a fair,
efficient and transparent securities market. Any person can get information

regarding the trading of various securities from the local terminals of NSE.
Thereby, it helps in reducing fraud in trading.
(iv) One of the objectives of NSE includes enabling shorter settlement cycles
and book entry settlements.
(v) NSE aimed at meeting the international standards and benchmarks of
stock exchange.
Answer needs Correction? Click Here
Q13 :
OTCEI was started on the lines of
(a) NASDAQ
(b) NYSE
(c) NASAQ
(d) NSE
Answer :
OTCEI (Over the Counter Exchange of India) was incorporated in the year
1990 on the lines of NASDAQ which is the OTC in USA. OTCEI is a fully
computerised and transparent stock exchange. It was established with the
objective of addressing the needs of small companies and helps in
maintaining the liquidity of their securities.
Answer needs Correction? Click Here
Q14 :
What is the OTCEI?
Answer :
Over the Counter Exchange of India (OTCEI) was incorporated in 1990 under
the Companies Act, 1956 and was recognised as a stock exchange under the
Securities Contracts Regulation Act, 1956. It commenced operations in the
year 1992. It aims at providing the small and medium companies an easy
access to the capital market. OTCEI is a fully computerised and single window
exchange system. OTCEI is modelled along the lines of NASDAQ, the OTC
exchange in USA. OTC was promoted by UTI, ICICI, IDBI, LIC, IFCI, GIC and SBI
financial services. It does not involve a geographical area rather, trading
takes place through its counters or offices through telephones and other

modes of communication. It acts as a place where buyers meet the sellers


and negotiate for an acceptable terms of trade. It provides a convenient,
transparent and efficient avenue for capital market investment. It
incorporates an exclusive list of companies as only those companies which
have an issued capital of 30 lakh or more can be listed on OTCEI. It provides
liquidity to the securities along with practicing a fair trade system. It also
aims at providing cheaper and easy means of trade to public as well as small
companies.
Answer needs Correction? Click Here
Q15 :
To be listed on OTCEI, the minimum capital requirement for a
company is
(a) Rs. 5 crores
(b) Rs. 3 crores
(c) Rs. 6 crores
(d) Rs. 1 crore
Answer :
To be listed on OTCEI, the minimum capital requirement for a company is Rs 3
crores and the maximum is Rs 50 crores.
Answer needs Correction? Click Here
<< Previous Chapter 1 : Financial ManagementNext Chapter 3 : Marketing >>

Chapter 11

Q1 :
What is marketing? What functions does it play with process of
exchange of goods and services? Explain.
Answer :
Marketing refers to the process wherein the buyers and sellers interact with
each other for purchase and sale of goods and services. Earlier, marketing
had different approaches with respect to its definition. It was sometimes
described as a post-production process that involves purchasing of the final
products and sometimes, as a pre-production process that involves
merchandising (designing) of the product. In reality, marketing is a much

wider concept than this. It consists of all those activities that are involved in
the process of exchange of the goods and services between producers and
consumers. These activities are basically the functions performed under
marketing. It involves planning, designing the product, packaging and
labelling of the product, standardising, branding, warehousing,
transportation, advertising, pricing and distribution. It also includes activities
that are performed even after the sale of product such as, maintaining
customer relations and collecting feedback. Thus, marketing plays an
important role in the process of exchange of goods and services.
Answer needs Correction? Click Here
Q2 :
Explain the advantages of branding to marketers of goods and
services.
Answer :
Branding implies giving a unique name, sign, symbol or term for the
identification of a product. The following are the advantages of branding to
the marketers.
i) Branding enables a firm to distinguish its product from the product of other
firms.
ii) It facilitates advertising of the product. Only when a product is given a
brand, it can be advertised and thereby, attract customers. A product with a
generic name cannot be advertised.
iii) Through good branding manufacturers can create loyalty and habituality
for their product. Thereby, the firm can benefit from this and charge a
different price for its product.
iv) It helps in establishing the base if a new product is to be launched. This is
because if a new product is launched under a good and established brand, it
is expected to get a good boost and benefit from reputation of the brand.
Answer needs Correction? Click Here
Q3 :
Distinguish between the product concept and production concept of
marketing.
Answer :

Basis of Difference

Product Concept

Belief

It was believed that consumers


favoured, superior quality products and
thereby, profits can be maximised by
increasing the quality of the product.

Focus of the business

Focus was on improving the quality of


the product, adding new features, etc.

Methodology

Production Concept

It was believed that consumers fav


readily available and affordable pr
thereby profits can be maximised
increasing the volume of productio

Focus was on enhancing the quant


production and reducing the avera
production.
Emphasis on improving the features and Emphasis on improving the produ
quality of product.
efficiency of the business.

Answer needs Correction? Click Here


Q4 :
List the characteristics of a good brand name.
Answer :
A good brand name consists of the following characteristics.
i) The name should be short and easy to spell, pronounce and remember. For
example, Ponds, Rin.
ii) A brand should suggest the benefits or qualities of the product and also
suit its functions. For example, Boost, Fair & Lovely.
iii) The name should be unique as well as adaptable to packaging and
labelling requirements. It should also be adaptable to different mediums of
advertising and languages.
iv) A brand name should be versatile and should be able to adapt to the new
products if introduced under the brand.
v) It must be distinctive and capable of being legally protected.
Answer needs Correction? Click Here
Q5 :
Product is a bundle of utilities? Do you agree? Comment.
Answer :

When a customer decides to buy a product, his/her main focus lies on the
utility which he/she receives while consuming it. A customer seeks different
types of satisfaction from the product. Benefits derived from a product can be
of three types- functional benefits, psychological benefits and social benefits.
For instance, when a consumer purchases a car, it provides him functional
utility as a means of transport. Besides, he also receives a psychological
benefit in the form of pride and self-esteem that he has bought a car. Along
with it comes the social benefit in the form of acceptance by the peers. Thus,
a product is said to be a bundle of utilities and a buyer while buying a product
values all such kinds of utilities.
Answer needs Correction? Click Here
Q6 :
What is the societal concept of marketing?
Answer :
According to the societal concept of marketing the organisations must
identify the needs of the market and the target consumers and deliver the
desired results in an efficient manner. The organisation should identify not
just the immediate needs of the market rather should also aim at long-term
well being of the consumers. Thus, besides customer satisfaction,
organisations should also aim at ecological, ethical and social aspects such
pollution, scarcity of resources, etc.
Answer needs Correction? Click Here
Q7 :
List the characteristics of convenience products.
Answer :
Convenience products refer to those products that are purchased frequently,
immediately and with minimum time and effort.
The following are the characteristics of a convenience product.
i) Such products are easily available at convenient places with minimum time
and effort wastage.
ii) Convenience products are consumed frequently and have a continuous
demand. Generally the essential commodities come under the category of
convenience products.

iii) They are available in small units and low and standardised prices.
iv) The competition in the market for such products is high. Thereby, heavy
advertising is required for these products.
Answer needs Correction? Click Here
Q8 :
Enlist the advantages of packaging of a customer product.
Answer :
Packaging refers to the process of developing and designing the container for
a product. A good packaging has the following advantages.
i) It enables differentiation and identification of a product from other
products.
ii) It acts as a promotional tool. Use of colours, pictures, symbols in packaging
helps in attracting the customers.
iii) Appropriate packaging contributes to the convenience in handling the
product.
iv) It helps in protecting the quality of the product from any kind of damage.
It helps in avoiding any kind of spoilage, breakage, etc. particularly at the
time of storage and transportation.
Answer needs Correction? Click Here
Q9 :
What are the limitations of advertising as a promotional tool? Enlist.
Answer :
The following are the limitations of advertising as a promotional tool.
i) It lacks personal form of communication and thereby is less forceful.
ii) Evaluation regarding the effectiveness of the advertisement is very difficult
to conduct.
iii) Advertisements come in standardised form and cannot be moulded as per
the requirements of different consumer groups.
iv) Effectiveness of advertising is low as there can be numerous
advertisement.

Answer needs Correction? Click Here


Q10 :
List five shopping products purchased by you or your family during
the last few months.
Answer :
Shopping products purchased are the following.
i) Clothes
ii) Jewellery
iii) Television
iv) Shoes
v) Home furniture
Answer needs Correction? Click Here
<< Previous Chapter 2 : Financial MarketsNext Chapter 4 : Consumer Protection >>

Q1 :
Define marketing. How is it different from selling? Discuss
Answer :
Marketing refers to the process wherein the buyers and sellers interact with
each other for purchase and sale of goods and services. It comprises of a
range of activities such as planning, designing the product, packaging and
labelling of the product, pricing and distribution and also after sale services
such as maintaining customer relations and collecting feedback. Selling, on
the other hand, refers to the promotion activities undertaken for the sale of
goods and services. Such promotion activities can be in the form of
advertising, publicity, etc. Through the process of selling the product is
converted into cash. In this sense, selling can be regarded as a part of
marketing.
The following points highlight the difference between marketing and selling.

Basis of Difference
Scope of the Term

Marketing
Marketing is a wider time and
involves a number of activities such
as planning, designing, after sale
services, etc.

Selling

Selling is a narrower term and is


confined just to the promotion activi
and the transfer of ownership of the
product from the seller to the purcha

Selling is just a part of marketing.


Means of profit
Maximisation

Marketing lays emphasis on


enhancing customer satisfaction for
maximising profits.

Selling lays emphasis on maximising


the sales for maximising profits.

Emphasis

All marketing activities emphasise on


customer satisfaction

Scope of Activities

Marketing activities start before


production and continues even after
the sale of the product.

Selling activities emphasise on sale o


goods and the transfer of ownership
product from the seller to the purcha
Selling activities start only after the
production of the product.

Strategies and Efforts

It involves strategies related to


production, planning, designing,
feedback services, etc.

It involves strategies related just to t


promotion and sale of the product.

Answer needs Correction? Click Here


Q2 :
What is marketing concept? How does it help in the effective
marketing of goods and services?
Answer :
Marketing concept of marketing management lays emphasis on customer
satisfaction. It believes that customer satisfaction plays a vital role in the
success of any organisation. In the long run, any organisation can survive and
maximise profits only if it identifies customer needs and effectively works
towards fulfilling them. This concept identifies the fact that people purchase a
product for satisfaction of a specific need (such as functional need, social
need, psychological need, etc.). Any organisation must aim towards
identifying such needs and satisfy them in an effective manner. That is, it
must take all decisions based on needs and requirements of the customers.
An organisation works and sells not according to what it has, but according to
what the customer wants. The marketing concept is based on the following
points.
i. The efforts of all marketing activities must be directed towards a particular
segment of market or group of customers.
ii. The organisations must clearly identify the needs and requirements of the
target customer.

iii. It should develop such products and services that satisfy the needs of the
customers.
iv. It should not just independently work towards customer satisfaction, but
should also aim at satisfying the customers better than its competitors.
v. The crux of all efforts of marketing is profit.
Marketing concept helps in effective marketing of goods and services. If all
marketing activities are directed towards customer satisfaction, marketing of
goods and services would take place in an effective and smooth manner. If
decisions of production, pricing, designing, etc. are based on the needs of the
customers, selling would not be a problem. For example, if customers want
dual sim mobile phones, high pixel camera, GPS, etc. the company produces
mobile phones with such features. Similarly, other decisions of the company
such as pricing, branding, etc. are also based on the requirements of the
customers.
Answer needs Correction? Click Here
Q3 :
What is marketing mix? What are its main elements? Explain.
Answer :
Marketing mix refers to the set of marketing tools that are used to achieve
the various objectives of marketing. In the process of marketing, market
offering plays an important role. That is, for effective marketing, an
organisation must decide the various features of a product such as its size,
quality, location of sale, etc. Such decisions are affected by a large number of
factors. Some of them are controllable by the firm. For example, decisions
regarding packing, branding, pricing, advertising, etc. are within the control
of the firm. However, there are certain non-controllable factors as well that
affect the decision making by a firm. For example, government policies, credit
policies of the banks are beyond the control of the firm. A firm continuously
alters the various controllable factors to achieve the objectives of marketing.
Such factors form the pillars of marketing and are known as marketing tools.
From the various alternative marketing tools, a firm chooses the best
combination to develop a market offering. Such a set of marketing tools used
by the firm to achieve the desired objectives of marketing is known as
marketing mix.
The following are the elements of marketing mix.

1. Product: A product refers to any good or service that offers value and
satisfies needs of a customer. For example, a car, toothpaste, soap, services
of teacher, etc. are products. In marketing, a product relates not just to the
physical product but it also includes the satisfaction of various needs and
utilities of the customer. For example, consumption of a product benefits a
consumer in the form of satisfaction of consumers' functional needs, social
needs and psychological needs. Such benefits also form a part of product. In
addition, a product also includes the after sale services such as taking
feedbacks, redressing consumer complaints, etc. Important decisions
regarding a product relate to its designing, quality, features, labelling,
branding and packaging.
2. Price: Price refers to the money paid by the customers to obtain a product.
Price of a product affects its demand. As the price of a product increases, its
demand falls and vice versa. The marketers must analyse properly the
various factors that determine the price and decide a suitable price for the
product. For example, the target customers, pricing policy followed by the
competitors, objectives of the firm, etc. must be consider while deciding a
price. The price set should be such that the customers find it at par with the
value of the product. Suitable discounts and incentive schemes must also be
decided.
3. Place: Besides the product decisions and pricing decisions, a firm must
also take decisions regarding the availability of the product to the target
customers. A firm must appropriately decide the dealers or intermediaries for
the distribution of the goods. Other important decisions comprise of
managing the inventory, warehousing, storage and transportation of the
goods
4. Promotion: Promotion comprises of those activities that communicate
customers regarding the availability of a product, its features, qualities, etc.
so as to influence the customers to purchase the product. Organisations
undertake various promotional activities such advertising, sales techniques,
personal selling, etc. An organisation must carefully decide the medium of
promotion, thereby, taking the related actions. For example, the organisation
must decide which of the sales techniques such as discounts, free gifts, sale,
etc. should be used.
Answer needs Correction? Click Here
Q4 :
What are industrial products? How are they different from consumer
products? Explain.

Answer :
Industrial products refer to those products that are used as inputs for the
production of other goods. Such goods are not meant for final consumption
rather they are used as raw material and inputs by the manufacturers for the
production of consumer goods. For example, machines, tools etc. are
industrial products. As against this, consumer products refer to those
products that are used by the ultimate customers for their personal
consumption purposes. For example, toothpaste, edible oil, furniture, etc. are
consumer goods. The following points highlight the difference between
industrial products and consumer products.

Basis of Difference
Number of Customers

Channel of Distribution

Industrial Products
The number of customers is limited. For
instance, oil seeds (industrial product)
are used mostly by the producers of
mustard oil.
Such products require shorter channels
of distribution such as direct selling or
one level channel.

Location

Industrial products remain concentrated


only in those areas where the industries
producing these goods are located.
Demand
Demand for industrial product is a
derived demand based on the demand
for consumer products.
Role of Technical Features Technical features play an important
in Decision Making
role while purchasing these products

Consumer Products

The number of customers is higher.


For instance, mustard oil (consumer
product) is consumed by many peop

Such products require comparatively


longer channels before they reach th
final consumer. However, channel o
distribution for perishable consumer
products is small.
Consumer products readily and
conveniently available.

Demand of consumer product is not


derived demand rather sets the basis
for demand for industrial products.
Such products do not involve any
technical complexities in
manufacturing. Thereby, technical
features does not have much role in
decision making while purchasing.

Answer needs Correction? Click Here


Q5 :
Distinguish between convenience product and shopping product.
Answer :

Basis of Difference

Convenience Product

Shopping Product

Demand

Convenience products have a continuous


and frequent demand.

Shopping products have a relati


frequent demand.

Nature of Products

Essential commodities come under the


category of convenience goods.
These products are available in small
units and have low unit price. Thereby,
such products have low profit margin.
Such products are brought impulsively
without devoting much time and effort.

Generally, such goods are durab


nature
These products usually come in
units and have high unit prices.
the profit margin is also high.
Such products are not brought
impulsively and the consumer d
considerable time and effort to c
the price, quality, etc. of the pro
Jewellery, furniture, clothes, etc

Unit of Purchase and Price


Nature of Purchase

Example

Ice-creams, medicines, newspaper,


stationary items.

Answer needs Correction? Click Here


Q6 :
What are the factors affecting the determination of the price of a
product or service?
Answer :
Price refers to the money paid by the customers to obtain a product and this
price affects its demand. Thereby, pricing plays an important role in the
marketing of goods. The price charged by a firm for its product affects it
revenue and profits as well. In addition to this, pricing also acts as a
competitive tool. Firms producing similar substitutable products compete with
each other on the basis of price. Thus, the firms must pay due emphasis on
proper pricing of their products. The marketers must analyse properly the
various factors that determine the price and decide a suitable price for the
product.
The following are the factors that affect the determination of the price of a
product or a service.
1. Cost of Product: Cost of the product plays an important role in
determining the price. It comprises of cost involved in production, distribution
and sale of the product. Cost of product can be classified into three broad
categories, namely, fixed cost, variable cost and semi variable cost. Fixed
cost refers to those cost that do not vary with the level of output produced.
For example, for the production of a good, a firm incurs cost on the purchase

of machinery, land, etc. Such costs are fixed cost. On the other hand, variable
cost refers to those costs that vary in direct proportion with the volume of
production. That is, as the level of output increases, the variable cost also
increase. For example, the cost incurred on labour, raw material, etc. are
variable cost. Semi variable cost refers to those costs that vary with the level
of output but not in direct proportion. For example, commission paid to
intermediaries for the sale of good is a semi-variable cost.
Generally, the firms decide the price of a product such that they are able to
cover all their cost. In addition, they also aim at earning some profit over and
above the cost incurred by them. Thus, the firms decide upon the price
keeping in consideration the cost as well as the profit factor.
2. Demand for the Product: While determining the price, a firm must also
consider the demand for its product. Herein, the elasticity of demand plays
an important role. Elasticity of demand refers to the proportionate change in
demand due to a given proportionate change in price. If due to small
proportionate a change in price, the demand changes by a larger proportion,
the demand is said to be elastic. That is, demand is said to be elastic, if a
small rise (or, fall) in price leads to a relatively large fall (or, rise) in price. In
this case, the firm cannot charge a higher price as it would lead to a large fall
in the demand. On the other hand, demand is said to be inelastic, if a change
in price does not affect the demand much. In this case, the firm has the
privilege of charging a higher price, as even at a higher price, the demand
would not fall much. Thus, price for goods having elastic demand is generally
lower than price for goods having inelastic demand.
3. Degree of Competition in the Market: Generally, higher the
competition in the market, the lower is the price that a firm can charge for its
product. This is because in case of high competition, if a firm attempts to
charge a high price, it would lose its customers to the competitors. On the
other hand, if a firm faces very little competition for its product then it has
the freedom of charging a higher price.
4. Government Regulations: At times to protect the interest of public at
large, the government intervenes in the determination of price. For example,
in case of essential commodities, the government can declare a maximum
price that can be charged.
5. Objectives of Pricing: Every firm has various pricing objectives which it
considers while deciding a price. The following are some of the objectives of
pricing.
i. Profit Maximisation: Every firm aims at profit maximisation. However, if the
firm aims at maximising profits only in the short run, then it may decide to

charge a higher price and increase its revenue. On the other hand, if the firm
aims to maximise profit in the long run, it would charge a lower price so as to
acquire a greater share of the market and benefit from larger sales.
ii. Acquiring Market Share: If a firm desires to capture a greater market share,
it would charge a lower price so as to attract a greater number of customers
towards its product.
iii. Surviving Competition: In face of high competition, a firm would keep the
price for its product lower. This is because if it charges a higher price, it would
lose its customers to the competitors.
iv. Focus on Quality: If the firm emphasis on enhancing the quality of the
product, it charges a higher price to cover the additional cost incurred.
6. Method of Marketing: Methods of marketing used by the firm such as
distribution, advertisement, customer services, branding, etc. also affect the
determination of prices. For example, if the firm uses intense advertising for
the promotion of the product, then it would charge a higher price.
Answer needs Correction? Click Here
Q7 :
'Products is a mixture of tangible and intangible attributes'. Discuss.
Answer :
Generally, a product is referred to as a tangible asset which has physical
attributes. For example, we regard a motorbike or a laptop as a product.
However, in marketing a product relates not just to physical or tangible
attributes but also to certain intangible attributes. That is, a consumers'
decision to purchase a product is based not just on its tangible attributes but
also on certain other intangible attributes. In other words, a consumer
purchases a product not just for its functional utility but for other factors as
well such as brand name, reputation, social satisfaction, etc. For example,
when a person purchases a laptop, along with the physical attributes and
functional utility regarding the model, size, features, etc. he also looks for
intangible attributes such as guarantee, brand, etc. Hence, product is a
mixture of tangible and intangible attributes.
Answer needs Correction? Click Here
Q8 :
What do you mean by channels of distribution? What functions do
they play in the distribution of goods and services? Explain.

Answer :
Channels of distribution refer to the individuals, institutions, agents who
facilitate the process of distribution. As the potential consumers of a product
are spread over a larger geographical area, it becomes difficult for the
producers or the manufacturers to directly contact the customers for the sale
of their products. Here, channels of distribution play an important role. They
facilitate the transfer of goods from the place of production to the place
where they are consumed. For example, for a manufacturer of sugar in
Punjab, it would be difficult to contact the customers in other parts of the
country. To ease the process, it would sell its product to whole sellers who in
turn would sell it to the retailers. The retailers then finally sell the product to
the customers. In addition, channels of distribution also reduce the efforts of
the consumers by offering various goods and services at a convenient single
location. For example, at a retail store a customer can get a wide variety of
goods.
Thus, channels of distribution refer to the set of individuals, agents and
institution that facilitate the exchange or transfer of goods and services from
the producer to the consumer.
The following are the functions of channels of distribution.
1. Arrangement: An intermediary receives the supply of goods from various
sources. However, the goods received differ in terms of size, quality and
features. The intermediary arranges or sorts these goods into homogeneous
groups based on their characteristics.
2. Collection: A middle men accumulates and maintains large stock of the
goods so as to ensure a continuous and smooth flow of supply.
3. Allocation and Packing: A middle men breaks the whole lot of goods into
small, marketable units. It repacks the goods into convenient packets.
4. Building Variety: An intermediary acquires various goods from different
sources and assembles them at a single place. Thus, it maintains a variety of
goods. For example, a grocer maintains a wide variety of products for sale.
5. Promotion of Product: They assist in the promotion activities undertaken
by the manufacturers. For example, the manufacturers use advertising for
the promotion of their product. The intermediaries can aid this process by
putting banners and displays.
6. Mediation: On one hand, the middle men interact with the producers and
on the other hand, with the customers. Thus, they form a link between the

producers and the customers. They negotiate on matters related to price,


quality, etc. and work towards satisfying the needs of both the parties.
7. Bearing Risk: Intermediaries acquire goods from the producers and keep
them in their possession till the final sale. In the process they bear the risk of
fluctuations in demand, price, spoilage, etc. For example, suppose a retailer
acquires large quantities of sugar. However, after a period of time, the price
of sugar rises which reduces its demand. Thereby, the retailer may lose out
as the stock remains unsold.
Answer needs Correction? Click Here
Q9 :
Describe the functions of labelling in the marketing of products.
Answer :
In the marketing process, labelling plays an important role in packaging of
the product. Labelling basically provides the information about the product in
the form of a tag (in cases of local unbranded products such as rice, wheat,
etc.) or graphics (such as a lady applying cream on face for a face-cream).
There are various functions performed by labelling which are as follows.
i) Description of Use and Contents
The main function performed by labelling is to provide the description about
the product regarding its use, application, cautions, content, etc. For
example, on the bottle of a shampoo, the contents are written, along with the
cautions such as keep away from children below the age of 10. Similarly, on
food products such as maggie cuppa mania, a picture of noodles is there
describing its appearance. Along with this, the ingredients and procedure of
making it is also stated.
ii) Identification and Differentiation
A label helps in easy identification of the product. It helps the consumers to
differentiate their favourable products from other products. For example, in a
potato chip product, a man eating potato chips is imprinted for identification.
A label also provides other valuable information such as name or address of
the manufacturer, net weight, maximum retail price, batch number, etc.
iii) Standardising and Grading
A label also helps in setting grades for a product. This helps the marketers to
classify the product in different categories based on certain specific quality or

features. For example, a brand of face-cream comes in different categories


such as oily skin, dry skin and normal skin.
iv) Promotes the Production
An attractive label helps the marketers in promoting the product as well. It
helps in attracting more customers towards the product. For example, for
maggie noodles, the label says ''Taste bhi, Health bhi''. Herein, label plays an
important role in highlighting the product and promoting its sale.
v) Information Required for Law
Label also provides and mentions the information which are required to be
mentioned as per the law. For example, on a packet of tobacco it is written
'chewing tobacco is injurious to health'.
Answer needs Correction? Click Here
Q10 :
Explain the major activities involved in the physical distribution of
products.
Answer :
Once the good are produced, packaged, promoted, branded and priced they
must be made available to the consumers. That is, they must be moved from
the place where they are produced to the place where they would be
consumed. This physical movement of products from the place of production
to the place of consumption is known as physical distribution of products. It
involves the following four major activities.
1. Processing of Order: Processing of order and delivery of goods is the
most important component of buyer-seller relationship. Processing of order
comprises of a number of a steps such as placement of order, transmission of
the order by the intermediaries to the manufacturer, maintenance of
inventory as per the requirement, delivery of goods, etc. All such processes
take time. Physical distribution system should be such that it should ensure
speedy and proper order processing. Generally, there exists a direct
relationship between speed and accuracy of order processing and customer
satisfaction. Fast and accurate order processing provides a consumer greater
satisfaction. Thus, a proper system of order processing is of vital importance.
2. Transportation of Products: Transportation of products refers to the
physical movement of goods from the place where they are manufactured to
the place where they are consumed. The potential consumers of a product
are spread across a large geographical area. To make the goods physically

available to them, they must be transported from the place of production to


the place of consumption. Unless the products are physically transported and
made available at the place of consumption, sale cannot take place.
3. Warehousing: Generally, there is a time lag between the act of
production and the act of sale or consumption. This implies that the goods
must be properly stored and assorted before the actual sale takes place. This
process of storing and assorting the products is known as warehousing.
Proper warehousing ensures efficiency in delivery and sale of products which
in turn results in greater customer satisfaction. For example, through
warehousing a company can maintain stock of products and ensure timely
delivery of goods as and when required. However, maintaining warehouses
involves its own cost. Thus, a company must weigh the relative benefits and
costs associated with warehousing and maintain a balance between the two
as per the requirement.
4. Maintenance of Inventory: Inventory implies maintaining a stock of
products. The firms maintain inventory so as to ensure timely supply of
products. Inventory shares a direct relation with customer service.
Maintenance of higher inventory helps a firm in ensuring timely delivery of
goods to the customers and thereby, improve customer service. However,
maintenance of inventory involves a cost as a huge amount of capital
remains tied up in the stock unless it is sold. Thus, the firms must strike a
balance between customer service and cost. A firms' decision to maintain
inventory is based on several factors such as how well the distribution system
responds to the orders and the deliveries, cost involved in holding the
inventory, firms' objectives, etc. An appropriate inventory must be decided by
the firm keeping in consideration the various costs and benefits involved.
Answer needs Correction? Click Here
Q11 :
Discuss the role of intermediaries in the distribution of consumer
non-durable products.
Answer :
Intermediaries play an important role in the distribution of consumer nondurables. They facilitate the transfer of goods from the place of production to
the place where they are consumed.
The following are the different functions performed by the intermediaries in
case of non-durables.

i) Arrangement: An intermediary receives the supply of goods from various


sources. He then sorts these goods into homogeneous groups based on their
characteristics such as size, quality, etc.
For instance, an electronic goods seller receives supply of different electronic
goods (T.V., washing machine etc.) and then sorts them based on their
functions.
ii) Collection: An intermediary maintains large stock of the goods so as to
ensure easy flow of supply. For instance, the electronic goods seller maintains
large stock of each type of the electronic item.
iii) Allocation and Packing: This function includes breaking the larger stock
into smaller units. For instance, each electronic item as well as their spare
parts are packed separately.
iv) Building Variety: An intermediary acquires various goods from different
sources and assembles them at a single place. Thus, it maintains a variety of
goods. He procures the products and then sells them in different
combinations as desired by the consumers. For instance, a television and a
video player are preferred together by most of the people. Thus, the retail
can sell a combination of both.
v) Promotion of Product: They assist in the promotion activities undertaken
by the manufacturers. For example, the manufacturers use advertising for
the promotion of their product. The intermediaries can aid this process by
putting banners and displays. For example, an electronic goods retailer puts
up banners for various products highlighting their features.
vi) Mediation: Middle men perform the function of setting a deal that can
satisfy both the producers and the consumers. They negotiate the price,
quality, quantity, etc. for efficient transfer of ownership so as to satisfy the
need of both the parties.
vii) Bearing Risk: Intermediaries acquire goods from the producers and keep
them in their possession till the final sale. In the process they bear the risk of
fluctuations in demand, price, spoilage, etc. For example, suppose a retailer
acquires large quantities of air conditioners. However, after a few months
winter sets in and the demand for air conditioners falls. Thus, the stock
remains unsold and retailer would suffer a loss.
Answer needs Correction? Click Here
Q12 :
Expenditure on advertising is a social waste' Do you agree? Discuss

Answer :
Expenditure on advertising is often considered as a social waste. Advertising
is criticised on the grounds that it unnecessarily adds to the cost of the
company, weakens social values and aggravates, builds up consumer needs
and desires for multiple products. However, proponents of advertising argue
that advertisement through greater sales brings down the cost and aids the
process of growth.
The following points help in judging whether advertising can be considered as
a waste.
1. Higher Cost: Advertisement expenses add to the cost of the company.
The companies in turn, pass this increased cost to the consumers in the form
of higher prices. However, the supporters of advertisement argue that
advertisement in fact brings down the per unit cost of production. This is
because through advertisement greater number of customers can be
attracted towards the product which in turn implies an increase in the
demand for the product. In response to the increased demand, the
manufacturers increase production. With increased production the per unit
cost of production comes down. Thus, it can be said, that even though
advertisement expenses add to the cost, this increase in cost can be
compensated through fall in the per unit cost due to increased demand and
production.
2. Weakens Social Values: One of the major criticisms to advertising is that
it weakens social values and instead promotes materialism in the society.
Advertisement attracts customers towards new products and induces them to
purchase it. With increased knowledge about the availability of new products,
the customers feel dissatisfied about what they currently have. In the
process, often they end up buying what they don't even require. However,
such a process of discontentment and purchase of new products is never
ending and materialism increases.
On the other hand, it is argued that advertisements just inform the buyers
about the availability of various products and the final decision to purchase
the product rests with the consumer. Advertisement increases the knowledge
of the customers and keeps them informed.
3. Creates Confusion: It is argued that a large number of advertisements on
similar products confuse the customers. For example, advertisements of
different hair oil claim for healthy and long hair by the use of their product.
With numerous advertisements the consumer gets confused as to the product
of which of the brand should be purchased. .

On the contrary, supporters of advertisement do not agree with this criticism


and argue that advertisement provided a choice to the consumers. The
consumers can make a rational choice for themselves after analysing various
factors such as price, style, quality, etc.
4. Promotes Inferior Goods: It is argued that products of both superior
quality as well as inferior quality are advertised. Through advertisement,
demand for inferior goods can also be induced. However, such a claim is only
partially true. This is because quality is a relative concept. What is inferior to
one consumer can be superior to another. Advertisements promote all kinds
of products and the consumer purchases them if it suits their requirements.
5. Objectionable Advertisements: It is said that often advertisement
undermine social values and are in bad taste. Sometimes the language,
images and the content of the advertisement may not appeal to the society
at large. For example, women in men cantered products (such as, shaving
creams, deodorants, etc.) are often criticised.
As against this, it is argued that good or bad taste is a subjective
phenomenon and varies from person to person. What may be acceptable by
one may be offensive for other.
Thus, it can be said that expenditure on advertisement though draws
criticism, but the objections are not entirely true. While on one hand, they
can be misused and have their own cons but on the other hand one cannot
deny the benefits derived from advertisement.
Answer needs Correction? Click Here
Q13 :
Explain the factors determining choice of channels of distribution.
Answer :
One of the important decisions of marketing involves the choice regarding
which channel of distribution to opt for. The following factors determine the
choice of channels.
i) Product Type
The choice of channel of distribution is based on the type of the product that
is produced. It is important to check whether the product is perishable or nonperishable, whether it is an industrial or a consumer product, whether its unit
value is high or low and also, the degree of complexity of the product. For
instance, if a good is perishable then short channels should be used rather
than the long ones. Similarly, if a product has a low unit value then longer

channel are preferred. In a similar manner, consumer products are distributed


through long channels while industrial products are distributed through short
channels.
ii) Characteristics of the Company
The two important characteristics of a company that affect the choice of
channel are its financial strength and the degree of control that the company
wishes to hold on the intermediaries. Shorter channels require greater funds
than longer channels and also offer greater control over the members of the
channel (intermediaries). Thus, companies that are financially strong or wish
to command greater control over the channel of distribution opt for shorter
channels of distribution.
iii) Competitive Factors
The degree of competition and the channels opted by other competitors
affect the choice of distribution channel. Depending on its policies a company
can adopt a similar channel as adopted by its competitors or opt for a
different channel. For example, if competitors of a company opt for sale
through retail store, it may also do the same or it can opt a different channel
such as direct selling.
iv) Environmental Factors
Environmental factors such as economic constraints and legal policies play an
important role in the choice of channel of distribution. For example,
requirement of complex legal formalities at each step of distribution induces
the companies to opt for shorter channels.
v) Market Factors
Various other factors such as size of the market, geographical concentration
of buyers, quantity demanded, etc. also affect the choice between the
channels. For instance, if potential buyers are concentrated in a small
geographical area then, shorter channels are used. As against this, if the
buyers are dispersed in a larger area then longer channels of distribution may
be used.
Answer needs Correction? Click Here
Q14 :
Distinguish between advertising and personal selling.
Answer :

Advertising is an impersonal, paid form of communication used by the


marketers for the promotion of goods and services. On the other hand,
personal selling involves direct communication of the seller with the potential
customers. That is, it involves direct face to face communication of the sellers
with the customers for the purpose of sale of the product. The following
points highlight the difference between advertising and personal selling.

Advertising

Basis of Difference
Personal v/s Impersonal

Reach
Flexibility
Target Group

Cost Involved
Time Involved

Customer Feedback
Medium of Communication
Objective

It is an impersonal form of communication


where the seller communicates with
customers through various medium such
as television, newspapers, etc.
Advertising has a broader reach as the
advertisement reaches a large number
people simultaneously.
It is inflexible as advertisements are
standardised and cannot be adjusted as per
the requirements of different customers.
It is more suitable where marketing is to
be done to large number of consumers. For
example, advertising is suitable if,
marketing is to be done for ultimate
customers.
As advertising reaches the masses
simultaneously, the cost per person is low.
Advertising reaches a large number of
people simultaneously. Thus, it can cover
the entire market in a short period of time.
Through advertising, feedbacks and
reactions of the customers cannot be
judged.
It involves communication through mass
media such as television, newspapers,
radio, etc.
The basic objective of advertising is to
create interest of the customers towards
the product.

Answer needs Correction? Click Here


Q15 :
Explain briefly the components of physical distribution.

Personal Selling

It is a personal form of commu


where the seller directly interac
the potential customers.

It has a narrower reach as only


people can be contacted directl

It is flexible as the seller can ad


message as per the requiremen
different customers.
It is more suitable when marke
be done for a few selected cons
For example, if marketing is to
for intermediaries and retailers
personal selling is more useful
Personal selling is relatively co

As through personal selling on


people can be contacted, it take
time and effort to cover the ent
market.
As the seller directly contacts t
customers, he get feedback fro
customers and judge their reac
It personal communication thro
sales persons.

The basic objective of persona


is to create awareness about th
and induce decision making.

Answer :
Physical distribution refers to movement of products from the place of
production to the place of consumption. The following are the components of
physical distribution.
i) Processing of Order: Processing of order comprises of a number of steps
such as placement of order, transmission of the order by the intermediaries
to the manufacturer, maintenance of inventory as per the requirement,
delivery of goods, etc. As all such processes take time, a physical distribution
system should be such that it should ensure speedy and proper order
processing. Generally, there exists a direct relationship between speed and
accuracy of order processing and customer satisfaction. Fast and accurate
order processing results in greater consumer satisfaction.
ii) Transportation of Products: Transportation of products refers to the
physical movement of goods from the place where they are manufactured to
the place where they are consumed. To make the goods physically available
to the consumers they must be transported from the place of production to
the place of consumption.
iii) Warehousing: Warehousing refers to the process of storing the produced
goods before the final act of sale. If a company has a larger number of
warehouses, it will be able to readily provide the goods on time at different
locations. However, maintaining warehouses involves its own cost. Thus, a
company must weigh the relative benefits and costs associated with
warehousing and maintain a balance between the two as per the
requirement.
iv) Maintenance of Inventory: The firms maintain inventory so as to ensure
timely supply of products. Similar to warehousing, maintenance of inventory
also shares a positive relation with customer service. However, maintenance
of inventory involves a cost as a huge amount of capital remains tied up in
the stock unless it is sold. Thus, the firms must strike a balance between
customer service and cost.
Answer needs Correction? Click Here
Q16 :
Define advertising. What are its main features? Explain.
Answer :

Advertising is a technique used for promotion of a product. Through


advertising the companies attract customers towards their product and
induce them to purchase it. Some of the common modes of advertising are
newspapers, magazines, television, etc. The following are the important
features of advertising.
i) Cost Involved: Advertising involves a cost. It is a paid form of promotion.
The costs involved in advertising are to be borne by the sponsors.
ii) Impersonal Mode: Advertising is an impersonal mode of communication.
That is, there is no face-to-face interaction between the customer and the
advertiser. Thus, it lacks a personal touch and creates a monologue.
iii) Specific Sponsor: There are always some identified individuals or
sponsors who undertake the responsibility of designing it and bearing the
cost involved.
Answer needs Correction? Click Here
Q17 :
Discuss the role of 'sales promotion' as an element of promotion
mix.
Answer :
Sales promotion refers to the incentives that are offered to the buyers so as
to encourage them to purchase the product. It included activities such as
offering discounts, gifts, free samples, etc. Such activities supplement other
promotional activities undertaken by the company such as advertising and
direct selling. They increasingly attract the customers and induce them to
immediately purchase the product. Such activities are especially useful at the
time of launch of a new product. They bring an initial boost to sales. For
example, discounts are given in the form of offering the product at a lower
price than the listed price. Free gifts are offered in the form of quantity gifts
(such as 20% extra or buy 2, get 1 free). Free samples are provided in the
form of small packets when launching a new brand. Such incentives are opted
by the companies to attract more customers and boost its sales.

Chapter 12

Q1 :
Explain the importance of consumer protection from the point of
view of a business.
Answer :
Consumer protection refers to the consumer education regarding their rights
and responsibilities and the redressal of their grievances and complaints.
Consumer protection is important not just for consumers but also for
business. The following points highlight the significance of consumer
protection from the point of view of business.
(i) Long-Term Interest: With increasing competition and the drive to survive
in the market, consumer satisfaction and interest is of prime importance for
any business. While taking due care of the interest of the consumers, a
business builds goodwill and reputation. A satisfied customer not only comes
back and results in repeated sales for the firm, but also pulls new customers
by spreading the good word. Thus, it is in long-term benefit of the business to
protect consumers' interest.
(ii) Use of Resources: A business uses the resources of society for the
production of various goods and services. Through the sale of these goods
and services to the society, a business earns profits. Thus, it is their duty to
supply such goods and services that adhere to the overall interest of the
society.
(iii) Ethical Reasoning: Today moral values and ethics play an important
role in business. It is the moral duty of every business to protect the interests
of its customers. They must follow ethical and moral values and avoid any
form of exploitation of the consumers such as unfair trade practices,
adulteration, etc.
(iv) Responsibility towards Society: As the business earns profits by
selling various goods and services to consumers, it becomes their
responsibility to take care of the interests and satisfaction of the consumers.

(v) Government Interference: A business organisation that indulges in any form of


exploitation of consumers such as unfair trade practices, adulteration, etc. draws intervention
from the government. Such government interference spoils the goodwill and image of the
business. Thus, it is in the interest of the business to voluntarily follow such practices that
take care of the interest of the consumers.
Answer needs Correction? Click Here
Q2 :

Explain the rights and responsibilities of a consumer.


Answer :
The Consumer Protection Act enlists six rights for a consumer. The following
are the rights of a consumer.
1. Right to Safety: Every consumer has the right to be safeguarded against
those goods and services that are hazardous to life, health and property. For
example, manufacturing defects in electrical appliances, cooking gas
cylinders can be injurious. The consumer has the right to be protected
against such products.
2. Right to be Informed: According to this, the consumer has the right to
be informed of the quality, quantity, price, ingredients, weight, etc. of the
goods and services. In India, it is legally mandatory for the manufactures to
provide all such information on the package and the label of the goods. This
enables the consumer to make a wise choice.
3. Right to Choose: A consumer has the right to choose from a variety of
goods and services at competitive prices as per his wishes. That is, the
consumer has the right to have access to a variety of products at fair and
competitive price. Thus, the retailer or the supplier should offer a variety of
products in terms of quality, brand, price, etc. They should not induce the
consumers towards the purchase of a particular product or service.
4. Right to be Heard: In case of any grievance or dissatisfaction, a
consumer has the right to file a complaint. A consumer can file a legal
complaint and seek redressal against any form of exploitation in appropriate
forums established by the government. Nowadays, many consumer
organisations and associations also work in this direction.
5. Right to Seek Redressal: A consumer has the right to seek redressal and
compensation in case of any exploitation. The Consumer Protection Act
provides for compensation in the form of replacement of product, cash
compensation, repair/removal of defects, etc.
6. Right to Consumer Education: A consumer has the right to be educated
and aware of his rights and the available remedies in case of exploitation.
Many government and non-government organisation work actively in this
regard.
In addition to the rights, a consumer has some responsibilities to safeguard
himself from exploitation. The following are some of the responsibilities of a
consumer.

(i) Awareness: A consumer should be well aware of the availability of various


goods and services. This enables him to make an informed and wise choice.
(ii) Looking for Quality Marks: A consumer must always look for the quality
certification marks before the purchase of goods such as ISI in case of
electrical goods, AGMARK in case of agricultural goods, etc.
(iii) Asking for Cash Memo: A consumer should always ask for cash memo
or bills for the goods and services purchased. The bill acts a proof of purchase
and can be used for future references in case of a complaint.
(iv) Reading Labels: Labels on goods provide information regarding price,
weight, expiry date of the product. The consumer must read these labels
carefully before purchasing the goods.
(v) Honest Transactions: A consumer must always choose legal dealings
and discourage illegal trade practices such as black marketing and hoarding.
(vi) Ensure Safety: A consumer must carefully read the manuals and
instructions provided by the manufacturer so as to ensure safe use of the
product. For example, instructions provided on the electrical appliances must
be carefully read before using them.
(vii) Filing Complaint: In case the product is found defective or any
deficiency is found in the quality of the product, a complaint must be filed in
the appropriate forum.
(viii) Consumer Societies: Consumer organisations and societies such as
consumer Coordination Council, Common Cause, Karnataka Consumer
Service Society, etc. must be formed to work towards consumer education
and awareness.
(ix) Environment Protection: The consumers must also work towards
environment protection by avoiding wastes, pollution control, etc.
Answer needs Correction? Click Here
Q3 :
Enumerate the various Acts passed by the Government of India
which help in the protection of consumers' interests.
Answer :
The following are some of the Acts passed by the Government of India for the
protection of consumers' interests.

(i) The Consumer Protection Act, 1986: This Act provides for the
protection of consumer interests against various forms of exploitation such as
defective goods, deficiency in services, unfair trade practices, etc. Under the
Act, a three-tier machinery comprising of District Forum, State Commissions
and the National Commission, has been set up for the redressal of consumer
grievances.
(ii) The Contract Act, 1982: This Act defines the conditions under which the
promises in a contract are binding. In addition, the remedies available in case
of breach of contract are also specified in the Act.
(iii) The Sale of Goods Act, 1930: The Act provides protection and relief to
the consumers in case the goods purchased by them do not abide by the
expressed or implied conditions and warranties.
(iv) The Agriculture Produce (Grading and Marking) Act, 1937: This Act
defines grade standards for agricultural and livestock commodities. It
provides the procedure for grading, packing and marking of agricultural
products. The quality certification mark provided by the Act is called as
AGMARK.
(v) The Prevention of Food Adulteration Act, 1954: This Act is formed to
check the adulteration in food items and to maintain their purity.
(vi) The Standards of Weights and Measures Act, 1976: This Act
safeguards the interest of the consumers against exploitative practices
related to under-weight and under-measure of the goods.
(vii) The Essential Commodities Act, 1955: This Act provides for control in
production, supply and distribution of essential goods. It also controls
inflationary trends and ensures equality in distribution of these goods. It also
checks anti-social activities such as hoarding and black marketing.
(viii) The Trade Marks Act, 1999: This Act is formed to check the use of
false marks on the goods and thereby, protects the consumers against such
goods.
(ix) The Competition Act, 2002: This Act is formed to prevent such
practices by the business firms that hinder competition in the market.
(x) The Bureau of Indian Standards Act, 1986: Under the Act, The Bureau
of Indian Standards has been established that formulates the standards for
the quality of the goods and based on these standards provides certification
of quality through BIS certification scheme. The quality certification mark
provided by the Act is called as ISI. In addition, a grievance cell has also been
set up to redress complaints regarding the quality of the products.

Answer needs Correction? Click Here


Q4 :
What are the various ways in which the objective of consumer
protection can be achieved? Explain the role of consumer
organisations and NGOs in this regard.
Answer :
Consumer protection refers to the making the consumers aware and
educated about their rights and responsibilities and help them in seeking
redressal for their grievances and complaints. Consumer protection is of
prime importance for both consumers and business. The following are some
of the ways in which the objective of consumer protection can be achieved.
1. Self Administration by Business: The business firms should realise that
it is in their long-term interest to protect the interest of their customers. They
must increasingly work towards their satisfaction. They must realise that
satisfied customers not only lead to repeated sales but also spread the good
word about the company and thereby, pull new customers for the business.
They must follow ethical values and avoid any form of exploitation of the
consumers such as unfair trade practices, adulteration, etc. In this regard,
nowadays many business firms have set up grievance cells and customer
care centres to redress the complaints of their customers.
2. Formation of Business Associations: In India, many business and
commerce associations such as Federation of Indian Chambers of Commerce
and industry (FICCI), Confederation of Indian Industries (CII) have been
established that work towards consumer protection. They lay down guidelines
for their members for the code of conduct for customer dealings.
3. Consumer Education and Awareness: Consumer education and
awareness is one of the important steps towards consumer protection. A
consumer must be well aware and informed of his rights as defined under the
Consumer Protection Act. He must also be aware of the reliefs available to
him in case of any form of exploitation under the Act. In addition, he must
also be educated about his responsibilities so as to safeguard himself against
cheating and exploitation. A well educated and informed consumer is able to
make wise choices.
4. Formation of Consumer Associations and Organisations: An
increasing number of consumer organisations and associations such as the
Consumer Unity and Trust Society, Consumer guidance Society of India,
Consumer Protection Council, etc. must be formed. These organisations work

actively towards educating the customers and making them aware of their
rights and responsibilities. In addition, they encourage consumers to protests
against any form of exploitation or cheating and also assists them in taking
legal actions and seeking redressal.
5. Role of Government: Government plays an important role in consumer
protection by enacting various rules and legislation in this regard. The
Government of India has passed various regulations and legislation to provide
for consumer protection such as The Consumer Protection Act, The Contract
Act, The Trade Marks Act, The Competition Act, etc. Of these, the most
important being the Consumer Protection Act. Under the Act a three-tier
machinery comprising of District Forum, State Commission and National
Commission has been set up that work towards the redressal of consumer
grievances and complaints.
Role of Consumer Organisations and NGOs in Consumer Protection
Nowadays, consumer organisations and NGOs play an important role in
protecting and promoting consumers' interest. Some of the important NGOs
and consumer organisations include Consumer Coordination Council,
Common Cause, Consumer Protection Council, Consumers' Association,
Mumbai Grahak Panchayat, etc. The following are some of the functions
performed by these organisations in regard of consumer protection.
(i) Consumer Education: The NGOs and consumer organisations organise
various training programmes and workshops to educate the consumer about
their rights and responsibilities.
(ii) Publishing Journals: In addition to the training programmes and
seminars, they also spread awareness through journals and periodicals.
These journals and other publications provide knowledge about various
consumer problems, legal remedies available and other such matters.
(iii) Legal Assistance: They also provide legal assistance and aid to the
consumers and help them in seeking suitable redressal.
(iv) Encouraging Protest against Exploitation: They encourage the
consumers to protest strongly against any form of exploitation and unfair
trade practices.
(v) Assistance in Filing Complaints: They encourage the consumers to file
complaints in appropriate forums and also file complaints on their behalf.
(vi) Taking Initiatives: They not only encourage the consumers to register
complaints but also take initiatives themselves in filing cases in the general
interest of the public.

(vii) Testing Quality of the Products: They carry out the quality tests for
various products in laboratories and publish the results. Such results help the
consumers in making informed choices.
Answer needs Correction? Click Here
Q5 :
What are the responsibilities of a consumer?
Answer :
A consumer is also responsible for safeguarding his own interest and to
protect himself from exploitation. The following are some of the
responsibilities of a consumer.
(i) Awareness: A consumer should be well aware of the availability of various
goods and services so that he can choose carefully and wisely.
(ii) Looking for Quality Marks: A consumer must always look for the quality
certification marks before the purchase of goods such as ISI in case of
electrical goods, AGMARK in case of agricultural goods, etc.
(iii) Asking for Cash Memo: A consumer should always ask for cash memo
or bills for the goods and services purchased. The bill acts a proof of purchase
and can be used for future references.
(iv) Reading Labels: Labels on goods provide information regarding price,
weight, expiry date of the product. The labels must be read carefully.
(v) Honest Transactions: A consumer must always choose legal dealings
and discourage illegal trade practices such as black marketing and hoarding.
(vi) Ensure Safety: A consumer must carefully read the manuals and
instructions provided by the manufacturer so as to ensure safe use of the
product.
(vii) Filing Complaint: In case the product is found defective or any
deficiency is found in the quality of the product, a complaint must be filed in
the appropriate forum.
(viii) Consumer Societies: Consumer organisations and societies must be
formed to work towards consumer education and awareness.
(ix) Environment Protection: The consumers must also work towards
environment protection by avoiding wastes, pollution, etc.
Answer needs Correction? Click Here

Q6 :
Explain the redressal mechanism available to the consumers under
the Consumer Protection Act.
Answer :
Under the Consumer Protection Act, a three-tier machinery has been set for
the redressal of consumer grievances and complaints. The machinery works
at the District level, State level and National level and are known as District
Consumer Dispute Redressal Forum (or District Forum), State Consumer
Dispute Redressal Commission (State Commission) and National Consumer
Dispute Redressal Commission (National Commission) respectively. The
following is a brief explanation of the machinery under the Consumer
Protection Act.
1. District Forum
District Forum is set up in each district by the concerned State Government.
It comprises of a president and two or more members, one of whom should
be a woman. A consumer can to file a complaint in a District forum in case
the value of goods in question, along with the compensation that is claimed is
less than Rs 20 lakh. As the Forum receives the complaint, it refers it to the
concerned party against whom the compliant is registered. After considering
the tests and reports and on hearing both the concerned parties, the forum
passes a judgement order. Moreover, in case the party filing the complaint is
not satisfied with the order, an appeal can be filed in the State Commission
within 30 days of passing the judgment.
2. State Commission
The State Government sets up State Commission for the redressal of
consumer grievances. Alike District Forum, State Commission also comprises
of a President and two or more members one of whom should be a woman. In
State Commission, a complaint can to be filed by a consumer in case the
value of goods /services in question along with the compensation claimed is
more than Rs 20 lakh but is less than Rs 1crore. After receiving the complaint,
the commission refers the complaint to the party against whom the compliant
is registered. If necessary, the sample of the good are sent for testing in the
laboratory. After considering the tests of the reports and on hearing the
concerned parties, the commission passes an order. An appeal can be filed
against the order of the Commission before the National commission within
30 days of judgement.
3. National Commission

The National Commission is set up by the Central Government. It consists of a


President and four or more members one of whom is a woman. A consumer
can file a complaint in the National Commission in case the value of goods in
question along with the compensation claimed is more than Rs 1 crore. After
referring to the sample proofs and on hearing the concerned party, the
commission passes an order. In case the aggrieved party is not satisfied with
the order, an appeal can be filed before the Supreme Court.
Available Reliefs
In case the concerned forum passes an order in favour of the aggrieved
party, it can pass one or more of the following directions to the opposite
party.
i. Repair the defective goods or remove the deficiency in the service.
ii. Replacement of the defective good with a new one.
iii. Refund the price paid by the consumer for the good or the service.
iv. Payment of compensation in cash for the loss or injury suffered.
v. Payment of punitive damages.
vi. Removal of misleading advertisement and issue of a correct
advertisement.
vii. Payment of an appropriate amount (not less than 5% of the good in
question) to be credited to Consumer Welfare Fund.

viii. Ceasing the manufacturing of hazardous goods.


Answer needs Correction? Click Here
Q7 :
Who can file a complaint in a consumer court?
Answer :
For the redressal of the grievances of consumers, consumer courts and
consumer forums have been established under the Consumer Protection Act.
The following can file a complaint in a consumer court.
(i) Any consumer
(ii) Any registered consumer association or organisation
(iii) The Central and State Government

(iv) A consumer or a group of consumers on behalf of a number of consumers


having same interest.
(v) A legal heir or a representative of the deceased consumer.
Answer needs Correction? Click Here
Q8 :
What kind of cases can be filed in a State Commission?
Answer :

State Commissions have been established under the Consumer Protection Act for the
redressal of various consumer grievances. It comprises of a president and one or more
members, one of whom is a women. A consumer can file a complaint in the appropriate State
Commission in case the value of the goods or the services in question is more than Rs 20
lakhs but is less than Rs 1crore. In addition, an aggrieved consumer not satisfied with the
judgement at the lower forum (District Forum) can also appeal at the State Commission.
Answer needs Correction? Click Here
Q9 :
Explain the role of consumer organisations and NGOs in protecting
and promoting consumers' interests.
Answer :
Consumer organisations and NGOs play an important role in protecting and
promoting consumers' interests. Some of the important NGOs and consumer
organisations include Consumer Coordination Council, Common Cause,
Consumer Protection Council, Consumers' Association, Mumbai Grahak
Panchayat, etc. The following are the functions performed by these
organisations in regard of safeguarding the interest of the consumers.
(i) Consumer Education: The NGOs and consumer organisations educate
the consumer about their rights through various training programmes and
workshops.
(ii) Publishing Journals: They publish journals and periodicals to spread
knowledge and awareness about various consumer problems, legal remedies
available and other such matters.
(iii) Legal Assistance: They also provide legal assistance to the consumers
and help them in seeking suitable redressal.

(iv) Encouraging Protest against Exploitation: They encourage the


consumers to protest against any form of exploitation and unfair trade
practices.
(v) Assistance in Filing Complaints: They encourage the consumers to file
complaints in appropriate forums and also file complaints on their behalf.
(vi) Taking Initiatives: They not only encourage the consumers to register
complaints but also take initiatives themselves in filing cases in the general
interest of the public.
(vii) Testing Quality of the Products: They carry out the quality tests for
various products in laboratories and publish the results.

Vous aimerez peut-être aussi